Categories
Exam Questions Harvard Principles Suggested Reading Syllabus Teaching Undergraduate

Harvard. Principles of Economics. Reading assignments, Exams, 1928

 

Partial course outlines from Harvard’s principles of economics course from 1927-28 and 1928-29 were found filed with the economics course outlines for 1938-39 in the Harvard Archives. The principal instructors for the courses in both years were Harold Hitchings Burbank and Edward Hastings Chamberlin, so combining the first semester outline from 1928-29 with the second semester outline from 1927-28 as transcribed below gives us a synthetic syllabus for the 1927-29 years. This post also includes enrollment figures for the two academic years as well as the corresponding semester final exams for the course. Links to the assigned textbooks have been added to complete the package.

____________________________

Course Announcement and Description

ECONOMICS
GENERAL STATEMENT

Course A is introductory to the other courses. It is intended to give a general survey of the subject for those who take but one course in Economics, and also to prepare for the further study of the subject in advanced courses. It may not be taken by Freshmen without the consent of the instructor. Students concentrating in Economics should elect Course A in their Sophomore year, except in unusual cases. History 1 or Government 1, or both of these courses, will usually be taken to advantage before Economics A…

INTRODUCTORY COURSES
Primarily for Undergraduates

A. Principles of Economics

Tu., Th., Sat., at 11. Professor [Harold Hitchings] Burbank, Dr. [Edward Hastings] Chamberlin, Dr. [Charles Holt] Taylor, and Messrs. [John Bever] Crane, [Melvin Gardner] de Chazeau, [Edgar Jerome] Johnson, [Delmar] Leighton, [Talcott] Parsons, [Carl Johann] Ratzlaff, [James Harold] Shoemaker, [Samuel Sommerville] Stratton, [John Phillip] Wernette, [Harry Dexter] White and [Earle Micajah] Winslow; with lectures on selected subjects by Professor [Frank William] Taussig and other Members of the Department.

Course A gives a general introduction to economic study, and a general view of Economics for those who have not further time to give to the subject. It undertakes an analysis of the present organization of industry, the mechanism of exchange, the determination of value, and the distribution of wealth.

The course is conducted entirely by oral discussion in sections. Taussig’s Principles of Economics is used as the basis of discussion.

Course A may not be taken by Freshmen without the consent of the instructor.

SourceOfficial Register of Harvard University, Vol. XXV, No. 29 (May 26, 1928). Division of History, Government, and Economics 1928-29, pp. 63-64.

____________________________

Enrollment in Economics A, 1928-29

[Economics] A. Professor Burbank and Dr. Chamberlin, Dr. Taylor and Messrs. Leighton, Stratton, Winslow, O.H. Taylor, E.J. Johnson, de Chazeau, Parsons, Wernette, H.D. White, and Ratzlaff, Crane and Shoemaker. — Principles of Economics.

Total 477: 55 Seniors, 127 Juniors, 242 Sophomores, 26 Freshmen, 27 Others.

Source: Harvard University. Report of the President of Harvard College, 1928-29, p. 71.

 

____________________________

EXHIBIT D
First Half

OUTLINE OF STUDY FOR ECONOMICS A
1928-29

Hubert D. Henderson. Supply and Demand. (New York: 1922).

D. H. Robertson. The Control of Industry (London: 1923).

Frank W. Taussig. Principles of Economics, Vol. I, 3rd edition, (New York: 1921).

Sept. 27
Sept. 29
Lecture.
Lecture.
Oct. 1 – 6 Taussig, Principles 1. Wealth and Labor.
2. Labor in Production.
3. Division of Labor and Development of Modern Industry.
Oct. 8 – 13


Robertson
4. Large Scale Production.
5. Capital.
6. Corporate Organization of Industry.
1 – 3. Control of Industry.
Oct. 15 – 20 Taussig

8. Exchange, Value, Price.
9. Value and Utility.
10. Market Value. Demand and Supply.
Oct. 22 – 27

17. Coinage.
18. Quantity.
19. Secs. 2, 3, 4: History of Prices.
Oct. 29 – Nov. 3

20. Bimetallism.
22. Changes in Prices.
23. Government Paper Money
Nov. 5 – 10
24. Banking and Medium of Exchange.
25. Banking Operations.
Nov. 12 – 17

27. Banking System of United States
28. Crises.
29. Panics.
Nov. 19 – 24

Hour Exam
30. Prices.
31. Reform.
Nov. 26 – Dec. 1


Henderson
Review 8, 9, 10.
12. Constant Cost.
13. Diminishing Returns.
Demand and Supply (Nov. 26 to Dec. 15).
Dec. 3 – 8 Taussig
14. Varying Cost.
15. Monopoly.
Dec. 10 – 15
Henderson:
16. Joint Cost and Joint Demand.
Ch. 5. Demand and Supply.
Dec. 17 – 22 Taussig 32. The Foreign Exchanges
RECESS Dec. 23 to Jan. 2
Reading Period Jan. 2 to 16  [No additional reading requirements]
Jan. 2 – 7 Taussig
33. International Payments.
34. International Trade.
Jan. 9 – 14
36. Protection.
37. Free Trade.
MIDYEARS:

Source:  Harvard University Archives. Syllabi, course outlines and reading lists in Economics, 1895-2003. Box 2; Folder “Economics, 1938-1939 [sic].”

____________________________

1928-29
HARVARD UNIVERSITY
ECONOMICS A
[Mid-Year Examination, 1929]

  1. Many business men are hoping for a period of rising prices; some financial writers are prophesying that it is inevitable. Assuming no change in our existing monetary and banking laws, what causes might lead to an increase in prices? How would such rising prices tend to affect the holders of various types of securities?
  2. “Some people argue that price is determined by cost of production; and yet they admit that producers with too high costs have to drop out. Thus it is clear that in reality a producer’s cost is determined by the price he can get, consequently price cannot be determined by cost of production.” Comment on this statement.
  3. What influence has the existence of joint cost upon the development of large scale production?
  4. It has been stated that with the Federal Reserve System in operation there will never be a recurrence in the United States of such (a) crises and (b) panics as occurred in 1893 and 1907. Do you agree?
  5. What attitude toward the tariff would you expect to be taken by a banker who has made large loans abroad, by a manufacturer of woolen cloth, by a professor of economics, by a Louisiana politician?
  6. Explain briefly:
    1. The principles of subsidiary coinage.
    2. The relation between markets and the division of labor.
    3. The distinction between consumers’ goods and producers’ goods.
    4. The significance of the following: “The plentifulness of money is in itself a matter of indifference.”

Source: Harvard University Archives. Mid-Year examinations, 1852-1943. Box 11, Bound volume: Examination Papers: Mid-Years 1929, Papers Printed for Mid-Year Examinations [in] History, New Testament, Government, Economics….Military Science, Naval Science. January-February, 1929.

____________________________

Enrollment in Economics A, 1927-28

[Economics] A. Professor Burbank and Dr. Chamberlin and Messrs. K.W. Bigelow, [Theodore John] Kreps, Stratton, Winslow, O.H. Taylor, E.J. Johnson, de Chazeau, Parsons, Wernette, H.D. White, and D.V. Brown, with lectures on selected subjects by Professor Taussig and other Members of the Department. — Principles of Economics.

Total 532: 61 Seniors, 165 Juniors, 258 Sophomores, 20 Freshmen, 28 Others.

Source: Harvard University. Report of the President of Harvard College, 1927-28, p. 74.

____________________________

OUTLINE OF ASSIGNMENTS FOR ECONOMICS A
1927-28, 2nd. Half year.

Thomas Nixon Carver. The Distribution of Wealth (New York: 1921).

Hubert D. Henderson. Supply and Demand. (New York: 1922).

D. H. Robertson. The Control of Industry (London: 1923).

Frank W. Taussig. Principles of Economics, 3rd edition, (New York: 1921). Volume I, Volume II.

Feb. 6

Feb. 11

Review
Value
Diminishing Returns
Carver:

Distribution of Wealth
Ch. I. Value
Ch. II. Diminishing Returns
Feb. 13

Feb. 18

Rent Carver:
Taussig:
V. Rent
Ch. 44. Rent (esp. Capitalization)
Ch. 43. Urban Site Rent
Feb. 20

Feb. 25

Interest Carver:
Taussig:
Ch. VI. Interest
Ch. 40. Interest
Feb. 27

Mar. 3

Wages Carver:
Taussig:
Ch. IV. Wages
Ch. 47. Social Stratification
Mar. 5

Mar. 10

Profits, Population Carver:
Taussig:
Ch. VII. Profits
Ch. 53. Population
Ch. 54. Population, continued
Mar. 12

Mar. 17

Inequality Taussig:


Ch. 7. Productiveness
Ch. 45. Monopoly
Ch. 51. Great Fortunes
Ch. 55. Inequality
Mar. 19

Mar. 24

Land, Risk, Labor, etc. Henderson:



Ch. VI. Land
Ch. VII. Risk Bearing Enterprise
Ch. VIII. Capital
Ch. IX. Labor
Ch. X. Real Costs of Production
Mar. 26

Mar. 31

Labor Taussig:

Ch. 56. Wages system
Ch. 57. Labor Unions
Ch. 58. Labor Legislation
Apr. 2

Apr. 7

Labor

Ch. 59. Industrial Peace
Ch. 60. Workmen’s Insurance
Ch. 61. Coöperation
RECESS April 8-14
Apr. 16

Apr. 21

Railways
Ch. 62. Railways
Ch. 63. Railway Problems, continued.
Apr. 23

Apr. 28

Public Ownership & Combinations
Ch. 64. Public Ownership & Control
Ch. 65 Combinations & Trusts
Apr. 30

May 5

Industry and Capitalism Robertson:


Review
Ch. V. Capitalism of Industry
Ch. VI. Finance and Industry
Ch. VII. Survey of CapitalismCh. X. Workers’ Control
May 7
READING PERIOD BEGINS
May 12
Socialism Taussig:
Ch. 66. Socialism
Ch. 67. Socialism, continued.
May 14

May 19

Social Reform Robertson:

Ch. IX. Collectivism
Ch. X. Workers Control
Ch. XI. Joint Control
May 21

May 26

Taxation

Taussig:

Ch. 68. Principles Underlying Taxation
Ch. 69 Income and Inheritance Taxes
REVIEW
EXAMINATIONS

Source:  Harvard University Archives. Syllabi, course outlines and reading lists in Economics, 1895-2003. Box 2; Folder “Economics, 1938-1939 [sic].”

____________________________

1927-28
HARVARD UNIVERSITY
ECONOMICS A
[Final End-year Examination]

Allow one hour and one-half for the first question.

  1. Explain how the distribution of wealth is affected by the following:
    1. Large and rapid changes in the supply of money.
    2. Labor saving inventions.
    3. A rise in the standard of living of the wage earning classes.
    4. The opening for settlement of new areas of good agricultural land.
    5. The government regulation of public utilities.
  2. Discuss the accuracy of the following statements:
    “Three generations from shirt sleeves to shirt sleeves.”
    “The rich are becoming richer and the poor poorer.”
    “To abolish wage slavery we must abolish the wages system; only through socialism can the wages system be forced to disappear.”
    “The one way a union can help its members is by limitation of the supply of hands.”
  3. What does each of the following propose: collectivism, single tax, producers’ coöperation, syndicalism?
  4. Explain briefly the case for and against minimum wage laws, unemployment insurance, progressive taxation of incomes, the restriction of immigration.

Source: Harvard University Archives. Examination papers, Finals (HUC 7000.28). Bound Volume 70 (1928). Papers Printed for Final Examinations [in] History, Church History,…Economics,…Military Science, Naval Science, June 1928.

Image Source: Harold Hitchings Burbank from Harvard Class Album 1934.

Categories
Curriculum Fields Harvard

Harvard. Mathematical Economics Recognized as Subfield of Theory. E.B. Wilson, Crum, and Schumpeter, 1933

 

What I find particularly striking in the following report of the Committee on Instruction in Mathematical Economics at Harvard (note the  first named of the trio is E. B. Wilson) is the forecast that economics graduate students will need to acquire tools of mathematical economics and statistics already in the mid 1930s because they will need them later, 1953-63, when they will be “at the height of their activity” and by which time (implicitly) the “rapidly increasing importance of theoretical and statistical work involving higher mathematics” will have caught up with them. I have appended the course names for the statistics and mathematics courses referred to by number in the report.

Related postings: 

_____________________

Meeting of the Committee (Wilson, Crum, Schumpeter) on
Instruction in the Mathematical Economics
Tuesday, May 9 [1933]

In view of the rapidly increasing importance of theoretical and statistical work involving higher mathematics, and of the possibility that a considerable number of economists may have to be adequately familiar with both mathematical theory and statistical procedure twenty to thirty years from now, that is, when many of our present students will be at the height of their activity, the Committee (Wilson, Crum, Schumpeter) agreed on the following recommendations to be submitted to the Department which they believe to be both necessary and sufficient in order to provide facilities for events to work in mathematical theory as applied to economics:

(1) Any student who may wish to do so should be allowed to offer mathematical economics as his special field within the requirements for the Ph.D. This would involve but a slight alteration of existing practice which permits students to choose some branch of economic theory as a special field. The committee’s suggestion is merely that mathematical economics should be added to the other special subjects in economic theory which a student may select.

It seems desirable, moreover, to permit that any such student may select mathematics or rather some branch of pure or applied mathematics in place of one of the two remaining fields he has to offer.

(2) Advanced work in mathematical economics should conform to modern tendencies by stressing equally the mathematical side of economic theory and mathematical statistics. No student who elects mathematical economics as his special field should be allowed to do the one without the other. Especially courses 31a and 32b should be required also from students mainly interested in pure theory.

(3) Work in the Department of Mathematics through Math 5 should be considered as the minimum requirement as to mathematical training. Credit should be given only for Math 5, but not for any of the still more elementary course preparatory to it, which most of the students taking up mathematical economics will have had anyhow in their undergraduate period.

(4) No further steps should be taken at present. It seems best to see what the response will be before attempting to organize a special graduate course. The mathematical aspect of our subject is being dealt with in some courses already, and any Ph.D. candidates who may present themselves in case the rules be altered as recommended could easily be taken care of individually.

*  *  *  *  *  *  *  *

Copy of Letter from Harold H. Burbank to Joseph Schumpeter

October 3, 1933

Dear Joe,

I have read and approved without qualification the report of the Committee on Instruction in Mathematical Economics.

I think this report should be brought before the Department on the evening of Tuesday, October 10.

Very sincerely yours,

Prof. J. A. Schumpeter
2 Scott Street

HHB:VS

*  *  *  *  *  *  *  *

Graduate Instruction in the Mathematical Economics
Department Vote, October 10, 1933

In view of the rapidly increasing importance of theoretical and statistical work involving higher mathematics, and of the possibility that a considerable number of economists may have to be adequately familiar with both mathematical theory and statistical procedure twenty to thirty years from now, that is, when many of our present students will be at the height of their activity, the Committee (Wilson, Crum, Schumpeter) agreed on the following recommendations to be submitted to the Department which they believe to be both necessary and sufficient in order to provide facilities for events to work in mathematical theory as applied to economics.

The Department voted to accept the recommendations stated as follows:

(1) Any student who may wish to do so should be allowed to offer mathematical economics as his special field within the requirements for the Ph.D. This would involve no alteration of existing practice, which permits students to choose some branch of economic theory as a special field. The committee’s suggestion is that mathematical economics should be admissible.

(2) Any students using mathematical economics as his special field should be allowed to offer some branch of pure or applied mathematics as an allied field.

Work in the Department of Mathematics through Math 5, or the equivalent, should be considered as the minimum requirement as to mathematical training. Credit should be given only for Math 5, but not for any more elementary course preparatory to it.

(3) Advanced work in mathematical economics should conform to modern tendencies by stressing equally the mathematical side of economic theory and mathematical statistics. Therefore courses 31a and 32b should be required of anyone in electing mathematical theory as his special field.

(4) No further steps need be taken at present. It seems best to see what the response will be before attempting to organize a special graduate course. Any individual cases calling for special attention can be dealt with, under the proposed regulation, as our courses now stand.

Source:  Harvard University Archives. Department of Economics, Correspondence and papers 1930-1961. (UAV349.11), Box 13.

_____________________

Statistics Courses offered in the Department of Economics
at Harvard, 1934-35

Economics 31a 1hf (formerly Economics 41a). Theory of Economic Statistics, I

Half-course (first half-year). Mon., Wed., Fri., at 9. Professor Crum and Asst. Professor Frickey.
Economics 1a, or its equivalent, is a prerequisite for this course.

Economics 31b 2hf (formerly Economics 41b). Theory of Economic Statistics, II

Half-course (second half-year). Mon., Wed., Fri., at 9. Professor Crum and Asst. Professor Frickey.
Economics 1a, or its equivalent, is a prerequisite for this course.

Economics 32b 2hf (formerly Economics 42). Foundations of Statistical Theory

Half-course (second half-year). Tu., Th., 3 to 4.30. Professor E. B. Wilson.
Economics 31and one year of Calculus are prerequisites for this course.

Source: Announcement of the Courses of Instruction offered by the Faculty of Arts and Sciences, 1933-34(second edition), Official Register of Harvard University, Vol. XXX, No. 39 (September 20, 1933), p. 128.

_____________________

Undergraduate Mathematics Courses
at Harvard, 1934-35

Mostly Freshmen

[Mathematics] A. Professors J. L Coolidge et al. — Analytic Geometry; Introduction to the Calculus.

Mostly Sophomores

[Mathematics] 2. Professors Graustein et al. — Differential and Integral Calculus; Analytic Geometry.

Mostly Juniors

[Mathematics] 5a1hf. Professor Morse. — Differential and Integral Calculus (advanced course), Part I

[Mathematics] 5a2hf. Professor Morse. — Differential and Integral Calculus (advanced course), Part II

 

Source: Harvard University. Report of the President of Harvard College, 1934-35, p. 86.

 

Images:  Left to right: William Leonard Crum, Joseph A. Schumpeter, Edwin Bidwell Wilson. From the 1934 (Crum) and 1939 (Schumpeter and Wilson) Harvard Class Albums.

 

 

Categories
Economists Gender Harvard Transcript

Harvard/Radcliffe. Economics PhD alumna and Wharton professor, Anne C. Bezanson, 1929

 

The materials in this post are presented in the opposite order that they were actually assembled. I began with three pieces of correspondence and a transcript of economics courses for a Radcliffe graduate who was ABD (= “all but dissertation”) and still interested in submitting a thesis more than a decade after her last course work at Harvard. The economics department chairman, Harold H. Burbank, made no fuss and we can see from the record that Annie Catherine Bezanson was indeed awarded an economics Ph.D. in 1929.

After I filled in the course titles and professors for her transcript, I then proceeded to gather biographical/career information for Bezanson. It of course did not take very long to discover that shortly after being awarded her Ph.D. she was promoted to a  professorship with tenure, the first woman to have cleared that professional hurdle at the University of Pennsylvania. What turned out to be more challenging was to find any photo whatsoever. Fortunately I stumbled upon a genealogical site that posted a picture of Anne Catherine Bezanson along with the obituary that begins the content portion of the post…

_______________________

Obituary from Bezansons of Nova Scotia

Died, Feb. 4, 1980, Dr. Anne Bezanson bur. Riverside Cemetery, Upper Stewiacke. Professor Emeritus, Wharton School of Finance & Commerce, U. of Pennsylvania, d… Hanover, Mass.

Born Mt. Dalhousie, N.S. daughter of the late John and Sarah (Creighton) Bezanson. Dr. Bezanson went to the United States in 1901, where she received her A.B. degree, A.M. & PhD. from Radcliffe…member of the Phi Beta Kappa…awarded an honourary doctor of science degree from University of British Columbia and from the University of Pennsylvania…served as Director of the Industrial Research Dept., Wharton School of Finance and Commerce; was professor at the Graduate School of the University of Pennsylvania…served on the staff of the U.S. Coal Commission..member of Conference of Price Research, advisor to the Social Services Research Project, Rockefeller Foundation…wrote numerous articles in various professional economic journals …member American Economic Associationn; Historical Society of Pennsylvania; Economic History Association., serving as President from 1946-1948; American Statistical Association; Econometrics Society; Vice-President Delta Chapter Phi Beta Kappa, University of Pennsylvania.

Source: From the Website: Bezansons in North America

_______________________

PIONEER IN ACADEMIC BUSINESS RESEARCH
ANNE BEZANSON, PROFESSOR

ANNE BEZANSON had not yet completed her PhD in economic history in 1921, yet she was about to make history herself. At Wharton, the young Canadian helped establish the first business school research center, the Industrial Research Unit (later known as Industrial Research Department or IRD), with Professor Joseph Willits. The founding marked Wharton’s shift toward becoming an academic business research hub — defining a new role for business schools that continues today.

Bezanson’s 1921 article on promotion practices became the first product of the IRD. Bezanson continued her practical research in the early 1920s, writing a series on personnel issues, focusing on turnover, worker amenities, and accident prevention.

Willits and Bezanson designed an ambitious research program to explore and help civilize industrial working conditions, with the goal of social change. In 1922, Bezanson and Willits spent a year studying the earnings of coal miners at the U.S. Coal Commission. Employer associations, government agencies, and international organizations continued to look to the IRD for timely and practical knowledge.

In 1929, Bezanson finished her Harvard PhD and became the first female faculty member of Penn’s Graduate School of Arts and Sciences. Under her leadership as co-director (which continued until 1945), the IRD had many women on its team and pursued research into the economic status of workers, revealing for the first time hard proof of the disparities in salaries and promotions for women and minorities across many industries.

Bezanson became the first woman to get full tenure at Penn, and in the 1930s sat on the National Bureau of Economic Research Price Conference. From 1939 to 1950 Bezanson was a part-time consultant at the Rockefeller Foundation, where she organized the first-ever roundtable on economic history in 1940. As a result of this involvement, Bezanson played a crucial role in the creation of the Economic History Association in the early 1940s, serving as president between 1946–1947. She died in 1980.

Source:  University of Pennsylvania. The Wharton School.Wharton Alumni Magazine, 125th Anniversary Issue (Spring 2007).

_______________________

Harvard/Radcliffe Academic Record

A.B. magna cum laude in economics.

 Source:  Report of the President of Radcliffe College for 1914-1915, pp. 10,13.

 

A.M. Annie Bezanson….Southvale, N.S. [Nova Scotia]

Source:   Report of the President of Radcliffe College for 1915-1916, p. 12.

 

June 1929 Doctor of Philosophy

Annie Catherine Bezanson, A.B. (Radcliffe College), 1915; A.M. (ibid.), 1916. Subject, Economics. Special Field, Labor Problems. Dissertation, Earnings and Working Opportunity in the Upholstery Weavers’ Trade.

Source: Report of the President of Radcliffe College 1928-29, p. 321.

_______________________

Economics Coursework

HARVARD UNIVERSITY
(Inter-Departmental Correspondence Sheet)

Cambridge, Massachusetts

Miss Anne Bezanson, A.B., Radcliffe 1915; A.M., 1916.

1911-12

Ec 1….B [Principles of Economics, Prof. Taussig et al.]
Ec 5….B, A- [Economics of Transportation, half course. Prof. Ripley]

1912-13

Ec 23….A- [Economic History of Europe to the Middle of the Eighteenth Century. Dr. Gray]

1913-14

Ec 11….B [Economic Theory. Prof. Taussig]
Ec 24….A [Topics in the Economic History of the Nineteenth CenturyProf. Gay]

1914-15

Ec 7….. [Theories of Distribution. Prof. Carver, Excused for Generals.]

1914-15

Ec 13….A [Statistics: Theory, Method and Practice. Asst. Prof. Day]
Ec 34….A [Problems of Labor. Prof. Ripley]
Ec 12….B+ [Scope and Methods of Economic Investigation. Half-course. Prof. Carver]
Ec 33….B [International Trade and Tariff Problems in the United States. Half-course. Prof. Taussig]
Ec 20….A- [Course of Research. Probably Economic History with Prof. Gay]
Ec 14….A [History and Literature of Economics to the year 1848. Prof. Bullock]

Source: Harvard University Archives. Department of Economics. Correspondence & Papers 1902-1950. Box 3.

_______________________

Handwritten letter from Bezanson to Burbank

January 2, 1928 [sic]

My Dear Prof. Burbank:

A long time ago, I talked with Professor Young, as well as Professors Carver and Gay about submitting one of my studies in part fulfillment of the requirement for a doctor’s thesis. This request is the result of the difficulty of leaving my present work to complete the study upon which I was at work from 1915 to 1918 on the Industrial Revolution in France. This month when I completed the first analysis of the Earnings of Tapestry Weavers, I sent it to Professor Gay with the hope that it would be, or could be, made acceptable to the Department of Economics.

All this discussion has been informal and, of course, unofficial. I am now writing to you for advice about the official steps: should I apply to the Dean of the Graduate School for permission to change the thesis subject? or should this request go from you? Do you advise such a request and if so can it be made without changing my field of concentration?

Briefly my difficulty is that though I passed the General Examination in October, 1916, I have since not completed the thesis and final examination requirements. A degree seems to have some value in promotion here. Yet, I am engaged on studies which I cannot drop and go back to a subject as remote as French conditions. Dean Gay has been in touch with the progress of Tapestry Earnings and I am acting upon his suggestion in asking for an opinion upon the possibility of offering that study as a thesis.

Very sincerely yours
[signed]
Anne Bezanson

Industrial Research Department
University of Pennsylvania
Philadelphia, Pa.

Source: Harvard University Archives. Department of Economics. Correspondence & Papers 1902-1950. Box 3.

_______________________

Copies of responses by H.H. Burbank to Bezanson

 

January 7, 1929

Miss Anne Bezanson,
Index Research Department,
University of Pennsylvania,
Philadelphia, Pa.

Dear Miss Bezanson:

I see no reason why the program which you have offered for the Ph.D. cannot be changed to allow you to present your study on “Earnings in the Upholstery Weavers Trade”.

There will be some red tape about it. I expect I shall have to secure the consent of the Dean of the Graduate School and of the Department, but I foresee no difficulties in either direction.

I will write you as soon as there is a definite decision.

One question that is certain to be raised is whether or not the research is entirely your own work or whether it was carried on by an organization. I should like to have your reply to this as soon as possible. Your preface throws some light on this. I note that you say: “All analysis and interpretation of material has been made by the Index Research Department”. Does this mean that your own work was strictly limited to the writing of the report in the preparation of the material on which the investigation was based?

Very sincerely,
[H.H. Burbank]

 

*  *  *  *  *  *  *  *  *  *  *

January 9, 1929

Miss Anne Bezanson,
Index Research Department,
University of Pennsylvania,
Philadelphia, Pa.

Dear Miss Bezanson:

This is more or less a continuation of the note I sent to you yesterday. Last evening I talked to the members of the Department regarding your request. I think something can be worked out for you without very much trouble.

For your General Examination you presented Theory, Statistics, International Trade, Labor, and American History, reserving Economic History as your special field. It is my guess that you have done very little indeed with the literature of the field of Economic History during the last ten years, and that to prepare this field for a special examination would involve an inordinate amount of work. Further, it would require quite a stretch of the imagination to include your study of “The Upholstery Weavers” as Economic History.

Would it not be more within your general field of interest to present Labor problems as the subject for intensive examination. In spite of the fact that you presented this subject in your General Examinations it could be included as a special field. By a stroke of good fortune the Department put into effect this fall a ruling whereby candidates for the PhD may present an honor grade in an approved course in lieu of an oral examination in a subject. Ordinarily you would be required to stand for examination in Economic History as well as in Labor Problems, but under this new ruling we are able to accept the grade of A in Economics 24 taken in 1915.

Briefly then, it is my suggestion that your special field be Labor Problems, within which the dissertation which you are now presenting naturally would fall.

Please let me know if this meets with your approval.

Very sincerely,
H. H. Burbank.

HHB:BR

 

Source: Harvard University Archives. Department of Economics. Correspondence & Papers 1902-1950. Box 3.

Image Source: Website Bezansons in North America.

 

 

 

Categories
Columbia Economists Harvard NBER Stanford

Columbia. Economics Ph.D. alumnus. Moses Abramovitz, 1939

 

 

The professional career of Moses Abramovitz shows what a blend of Harvard and Columbia training in economics crowned by an NBER post-doc could get you back in the day. His contributions to the study of long-term growth and to the Stanford economics department’s rise to prominence are truly important legacies.

The first item of the post gives us Abramovitz’s personal quarter-century report to his Harvard classmates of 1932. This is followed by excerpts from Abramovitz’s memoir for his family that provide a rich account of his economics training at Harvard and then Columbia. A link to download the entire memoir is provided below. The post closes with a memorial resolution written by Abramovitz’s Stanford colleagues. But the real treat, is found in Moses Abramovitz’s description of his economics education and economists important for his development. Among other things we learn, the chairman of the Harvard economics department, Harold Burbank, was indeed anti-Semitic enough for Abramovitz not to have dignified him by name. Also we learn that in 1934 “Milton [Friedman] was much less ideological then than he later became, so he was a very pleasant and agreeable companion.”

_______________________

From the 25th reunion report of the Harvard Class of 1932

MOSES ABRAMOVITZ

Home address: 543 W. Crescent Drive, Palo Alto, Calif.
Office address: Dept. of Economics, Stanford University, Stanford, Calif.
Born: Jan. 1, 1912, Brooklyn, N.Y.
Parents: Nathan Abramovitz, Betty Goldenberg.
Prepared at: Erasmus Hall High School, Brooklyn, N.Y.
Years in College: 1928-1932.
Degrees: A.B. summa cum laude, 1932; Ph.D. (Columbia Univ.), 1939.
Married: Carrie Glasser, June 13, 1937, Brooklyn, N.Y.
Child: Joel Nathan, July 19, 1950.
Occupation: Professor of economics, Stanford University; member research staff, national Bureau of Economic Research.
Offices Held: Member editorial board, American Economic Review, 1951-54.
Member of: American Economic Association; American Statistical Association; American Economic History Association; Royal Economic Society; American Association for the Advancement of Science.
Publications: Price Theory for a Changing Economy; Inventories and Business Cycles; The Economics of Growth; “Capital Formation and Economic Growth,” editor; The Growth of Public Employment in Great Britain (with Vera Eliasberg).

I LEFT Harvard supported by a Sheldon Fellowship and exhilarated by the prospect of a year in Europe—no small piece of luck at any time and a pot of good fortune in 1932. Together with Dave Popper, I saw Paris and the Rhine country as they were before the second deluge. We saw our first Storm Trooper rallies in Heidelberg and, if we were not too innocent, we were certainly too full of good spirits to be greatly disturbed. But those charming days were suddenly cut short. From Nuremberg, I was called home by my father’s death.

Back in New York I began graduate work in economics at Columbia and continued there until 1935. In 1936, I was lucky enough to be brought back to Harvard as an instructor for two years and had the fun and satisfaction of being again in Cambridge as a teacher while my memories of life at college were still warm. At Columbia I had met another young economist whom I had known years before. I shall stick to the essentials. The young economist was a woman. We were married in 1937, so Carrie has had a year at Harvard, too.

In 1938, we were back in New York again, this time to work at the National Bureau of Economic Research. In the years that followed I learned what I know about scientific investigation from Wesley Mitchell and Arthur F. Burns. Together they were in the midst of their wide-ranging investigation of business cycles. They set me to work studying inventory fluctuations. In the fullness of time I got some results and published a book, a hefty volume called Inventories and Business Cycles. It got some notice and caused some controversy, and a certain number of copies continue to serve as ballast for bookcases that might otherwise be disturbed by a fresh breeze.

Early in 1942, I went to Washington to help Bob Nathan and the W.P.B. Planning Committee, first to goad the military into laying out programs big enough to make use of a national productive capacity they could not believe existed, and then to keep them from losing the munitions they really needed under the load of programs too large for even our capacity. A year later I was at O.S.S. working for Professor Langer and Dean Mason on German economic intelligence. My particular job was probably of little use during the war itself, but it produced a collection of materials and a few more or less knowledgeable individuals, and both were needed after the German defeat. I became involved in the negotiations about German reparations and in that way came to see Moscow in the months right after V-E Day. Our work, as we all now know, foundered in the general wreck of American-Soviet relations. Together with many other stalemated delegations on many other subjects, ours eventually came to Potsdam to be witnesses at the beginning of the partition of Germany and Europe.

Since 1948 I have been a professor at Stanford. We have one child, a boy now six. We think living here near San Francisco as comfortable and delightful as it can be; so I rush back east as often as I can to disgorge the lotus and discharge my guilt.

My chief activity is still, as it has been for many years, research in economics—a stubborn, unyielding, frustrating and altogether exasperating subject from which I don’t know how to shake loose. What do I believe? One’s bent of mind is shaped by one’s work. Mine is inclined to skepticism, not beliefs, still less belief. Very likely I have much to learn. Oh yes! I believe both parties are right – in what each says about the other.

Source:  Harvard Class of 1932, Twenty-fifth Anniversary Report (1957), pp.6-8.

_______________________

Undergraduate and graduate student days: memories of Harvard and Columbia

…My fourth course [freshman year at Harvard] was different. It was elementary economics. I was lucky. I drew an excellent instructor named Bigelow. Using Frank W. Taussig’s Principles, he introduced us to the general logic of the neoclassical theories of relative prices of commodities and of the factors of production, land, labor, and capital, to the distribution of income among these primary factors, to the theory of international trade, and to the virtues of free markets. He offered us a list of supplementary readings, one of which was called simply Supply and Demand, by an English economist, H.D. Henderson. It was a thin book, but it was a notable example of the lucid presentation of the logic of the economics of value and distribution. One could see all around one examples in ordinary life of the validity and importance of the theory. The way in which the various parts of the subject hung together in an interdependent system seemed not only analytically deep; it emerged as a beautiful structure, an aesthetic as well as a logical and tested structure. More than any other experience, it was this little book that drew me to go on with economics. When I returned to Harvard in September 1929, therefore, I chose economics as my field of concentration. And, indeed, when the economy began its collapse in October of that year, it confirmed me in my choice. It was a decisive experience.

Concentrating in Economics

Having chosen to concentrate in economics, I was assigned a tutor. Here again I was lucky. He was Edward S. Mason, then a still young assistant professor. But he was destined for both academic leadership and, as my story unfolds, for a real influence on practical affairs. Even more important for me, however, was the fact that this young man was already recognizably “wise,” a man of good judgment in both scholarly decisions and practical matters. He took a liking to me, and he remembered his friends! He was due to turn up with support and help at several critical junctures in my story.

My very first meeting with Mason was an exciting moment. It was late September or early October in 1929, that fateful year. We chatted, and then, more brash than usual, I said, “Well, Professor, when is the stock market going to break?” He answered, without hesitation, “Almost immediately.” And when I returned for our second meeting, it had happened. And then, still brash, I said, “Well, Professor, you must have made a mint of money.” And then I learned something about him and perhaps most academics of the time. He said, “Are you crazy? I have never owned a share of stocks in my life.”

… Like many, but not all, of the young economists of the time, who had no deep commitment to mainstream economics, I saw clearly enough that mainstream theory offered us no guidance in understanding the Great Contraction and Depression, and it was consequently a poor basis for public policy. Something new was needed, a theory that dealt more adequately with recurrent recessions and expansions of business and particularly with the very serious depressions and eventual recoveries which in the U.S. had succeeded one another at intervals of about 15 to 20 years since the 1830s. For the moment, I did not get beyond dissatisfaction with the older wisdom, Real enlightenment came only in 1936 with the publication of J.M. Keynes’s General Theory of Employment, Interest and Money. When I had absorbed Keynes’s reasoning, I became an enthusiasticKeynesian and I remain so to this day.

There was also a quite personal effect of these developments on my own work history. They prepared me to join the National Bureau of Economic Research when the chance came in 1937 and to do empirical research on business cycles under the direction of Wesley Mitchell and Arthur Burns, the most notable people doing such work at that time.

Still an undergraduate in 1929, however, at the beginning of the economic contraction and depression, I still had three years of undergraduate work to do. Guided by Mason and later by Douglas V. Brown, I took Taussig’s famous course in price theory at both the undergraduate and graduate levels. Taussig was then the leading American price theorist of his time and by far the most influential person in the Economics Department. In these courses, conducted by Socratic methods, he clearly formed a good opinion about me. I am sure he was of help to me behind the scenes at several junctures. I also remember two enlightening courses, Sumner Slichter on Labor Economics and John Williams on Money and Banking. In Williams’s course, I read Keynes’s earlier books and began to become familiar with his way of thinking. Anyhow, I did well in all these courses and in others in economics, history, and in one really interesting course in literature. That was Irving Babbett on Rousseau and Romanticism. I was apparently a natural-born good student and exam taker. The upshot was that I was graduated summa cum laude and I was given a Sheldon Traveling Fellowship.

For me, this last was more than an honor and more than a year of support and European travel and study at a time when money was so scarce and jobs for new college graduates almost nonexistent. My tutors and professors, including the influential Taussig, had already been encouraging me to think about going on to graduate study in economics and to an eventual academic career. To my parents and my brother, such a course was strange and uncertain. Abe began to call me “meshugana Moishele.” But it was clear that in the end they would support me in any decision I made. And the fellowship, which was tangible proof of the good opinion of the Harvard faculty, confirmed me in a career choice I had already more than half made: It was a decisive event.

[late June of 1932 left for Europe but Moses Abramovitz’s father died in September 1932]

… I resigned my scholarship and in that September of 1932 walked along Nostrand Avenue to Eastern Parkway and took the subway (IRT, Broadway and 7th Avenue Line) to Broadway and 116th Street. Half a block away, one entered Columbia. I walked in and registered and began three years of graduate work in economics. This was a big departure from the program I had thought lay before me, but I cannot remember any feeling of distress or resistance. I was glad to provide some degree of solid continuity for my mother, and I felt confident about the future. Columbia would also be a good start.

 

Columbia as a School of Economics

By forgoing Vienna, Cambridge, and Harvard, I had made a bigger change than I realized when I started in Columbia. Vienna, Cambridge, and Harvard were all centers in which understanding of the domestic economy of a country and of its international economic relations was squarely based on theoretical economics. This, in turn, was a doctrine logically derived from certain basic primary assumptions: that economic agents (consumers, savers, business firms, investors generally) were well informed, foresighted, and rational, and acted to promote their own individual interests, that they faced competitive markets and, as business firms, acted under the pressures of competition; they operated subject to the constraints of income and wealth and of market prices which they could not by their own actions significantly influence. Actions in this context were perceived as leading to an equilibrium of prices, wages, profits, etc., and of consumer satisfactions in which change might be harmful to some but would be more than offset by benefit to others. Thus, there was no room or occasion for public action except such as was necessary to enforce contracts, maintain competition, prevent or punish fraud and generally keep the peace. Changes in technology and in consumer tastes would lead to a new equilibrium of prices, rewards, incomes, etc., but such changes were viewed as “exogenous,” not the result of economic action or motivation and beyond the ken of economics.

The Columbia economists, however, rejected this structure of theory or, at least, its general application. They conceded its usefulness in explaining very simple matters: why a grand piano cost more than a pair of shoes, and, in general, why there is a rough association between the prices of commodities and their costs of production. They were skeptical, however, about the theoretical assumptions that agents were foresighted, well-informed, and rational. They saw markets as characterized by various degrees of monopoly power, with business firms capable not only of profiting by constraining production and raising prices more than costs alone would justify; they also often had the power to shape consumer tastes, for example by advertising, and, most important, to invest in research and development and so to advance and sometimes to retard—technological progress. They tended to see the economy as a whole, not as tending to an equilibrium, but as generating long-term growth of productivity, income, and wealth. This tendency did not, however, emerge continuously and at a stable rate but subject to recurrent fluctuations, loosely called “cyclical,” in which advance was sometimes fast,sometimes slow, and sometimes negative.

As I absorbed all this, I saw the justice of the Columbia outlook and came to appreciate its radical departure from the economics in which I had been trained as a Harvard undergraduate. Columbia economics, as it stood in the Thirties, however, had its own serious limitations. It was well advanced in its understanding of two subjects. One was in the study of the behavior of firms that had acquired and enjoyed various kinds and degrees of monopoly power. This was the province of Arthur Robert (“Columbia”) Burns—not the Arthur Frank (“Bureau”) Burns with whom I later did research on business cycles.

The other subject was another sphere of monopoly power, that of labor unions. Why were they so much less important in the U.S.A. than in Europe? What activities were successfully unionized and which not? And why? This was the area over which Leo Wolman ruled. Wolman later played a considerable role in the Roosevelt Administration, especially in connection with the disorders in the labor market stemming from the organizing drives of the AFL/CIO. He worked as chairman of the Automobile Labor Board, where he tried to keep the peace in that important industry—an effort that won him no friends in the unions. Wolman’s teaching, however, was as far from academic as can be imagined. It came directly from his own experience with labor unions. Although a professor at Columbia, he also worked as the economic advisor of Sidney Hillman, the president of the Amalgamated Clothing Workers, the men’s clothing union. Wolman learned as much as he advised. He saw clearly that in the flexible and mobile population conditions of the American continent, the only unions that could exercise strong and stable monopoly power were those operating in industries frozen in location. The newsprint industry was an example. The book print industry was not. Where the industry could move, it could flee from a union whose wage and other demands were excessive. Such a condition faced the Amalgamated, and Wolman used his influence to restrain labor’s demands. Even so, the industry moved from New York City to upstate New York, then down South, then to Chicago and on to California. It was the barrier to movement posed by small nation-states that made European unions stronger and more stable than America’s.

These subjects then were well taught at Columbia, and I felt I learned much from A.R. Burns and Leo Wolman. The basic academic tone of the faculty, however, stemmed from Wesley Mitchell. He had been the dominating influence on the faculty since he joined it just before the First World War. According to Mitchell’s own view of himself, his outlook stemmed in part from his early Midwestern origins. He was the son of a physician who was a small town practitioner in central Illinois. The down-to-earth pragmatism of the neighboring family farmers ran strongly in his personality. It was quite natural, therefore, that he should have been drawn to the philosophical schools of William James and John Dewey when these became prominent. Experience, not the logical implications of some generalized ideal, had to be our guide to life. He told about teasing his good Baptist grandmother and her conception of a God of Love who could yet condemn unbaptized infants to the torments of Hell.

[…]

Mitchell carried out his scheme and reported his findings, together with his evidence, in a large book with the simple title, Business Cycles. The book began with a summary of earlier work relevant to the subject together with the “speculations” (one of Mitchell’s favorite characterizations of largely theoretical but inadequately verified ideas). He used these as suggestions of subjects needing investigation. There followed Mitchell’s own quantitative studies of these and other subjects: production (agricultural and other), income, sales, retail, wholesale, manufacturing, etc., commodity prices, the prices of stocks and bonds, and the profits and interest rates they paid. Mitchell’s quantitative descriptions involved tracing the fluctuations of the behavior in these activities and of their long-term trend and seasonal fluctuations so that the fluctuations connected with business cycles could be seen free of the influence of trends and seasonal factors. The book ended with a statement of Mitchell’s views of how the concatenation of the behavior of the separate activities led to expansions of business activities in general followed by similarly general contractions, which in turn produced the conditions that generated another business expansion.

Mitchell’s book made a notable impression on economists. This was partly because now, for the first time, students of economics could base their attempts to explain business cycles and to develop a theoretical model based on definite quantitative information about the typical behavior of the major business activities. But it was partly, perhaps mainly, because it gave economists at large a new vision of how economic research could be carried on. It need not mainly consist of logical deductions from a set of preannounced assumptions. It could instead take the form of observed behavior, together with empirical tests of the hypotheses so formed based on fresh observations independent of those from which the hypotheses originally proposed had been drawn. It was this vision of an empirically based economics that was the spirit of the Columbia program, and it stood in sharp contrast to the program at Harvard, where I was introduced to the subject, and, indeed, with the economics then taught in the other leading universities.

I did not give up my allegiance to Harvard easily. Two episodes illustrate my resistance. Mitchell gave a course on business cycles. I chose to take it. It was a course that, in a sense, was a duplicate of his 1913 book, refreshed by data not available in 1913. But as I listened to Mitchell’s “analysis” of one time series after another—amplitude, lead or lag relative to the “reference” peak or trough (that is, relative to the peak or trough of the general business cycle), rates of expansion or contraction in successive thirds of the fluctuations, and more—I could make nothing of it. After some weeks I dropped the course. Mitchell signed the necessary form without demur and, apparently, never held it against me—a characteristic of his liberal and tolerant attitude.

In other respects, my year was pleasant and rewarding. I found Eli Ginzberg and began a lifelong friendship, the closest and most intimate in my life. Like other graduate students, I occupied a “cubicle” on the top floor of the new Butler Library—just enough space for a table, chair, and file cabinet. A friend said: “It’s all right if I am in there alone, but if I get an idea, I have to move into the corridor.” One day, there was a knock on my door, and in walked Eli. He had just returned from a scholarship, traveling the country and interviewing business executives, union bosses, politicians, etc. On his return, he asked Mrs. Stewart, the all-knowing department secretary, what new people were interesting. She mentioned me, and there he was. He sat down and began to tell me about his travels, the first of many sessions on the same subject.

One early reward of my new friendship was to come to know his parents. They occupied an eighth-floor apartment on 114th Street, directly behind the Butler Library. Eli’s father, Louis Ginzberg, was a professor in the Jewish Theological Seminary at 120th Street. He was perhaps the most notable Jewish scholar of his time, a specialist in Talmudic history and interpretation based on a wide knowledge of ancient Middle Eastern languages and in the history of its peoples. Eli began to bring me to their Friday evening suppers. I found old Louis to be a wise and humorous man, a fine companion and host for a pleasant evening.

On one of my first visits, Eli took me into Louis’s study to show me a lampshade that one of Louis’s students had made. The parchment shade was decorated. All around the shade were drawn the spines of books, and on each spine there appeared the title of one of Louis’s books, perhaps 14 or 15 in all. And then the student had an inspiration. He added one more spine and on it drew the title of Eli’s first book, his Ph.D. dissertation, The House of Adam Smith. At the time, we wondered whether Eli could duplicate his Father’s achievement. In fact, he did so many times over, in quantity at least, if not always in depth—something to which Eli did not aspire.

[…]

Now back to my struggle between Harvard and Columbia economics. In that second year at Columbia, the internal conflict found two new exponents. On the Columbia side was Eli. He was someone of great personal interest to me, but as an economist, he was an eccentric. He was a skeptic about anything theoretical and served mainly as an exemplar of Columbia’s tolerance for talent in whatever way it showed itself. On the Harvard side, there now appeared a powerful supporter. He was Milton Friedman, who had come to Columbia on a scholarship for a year of graduate work. We soon became good friends. It emerged that we two were the only Columbia students who had had a real training in neoclassical price theory, the very bedrock of the economics of the time. The faculty, moreover, refused to sanction a course in the subject, and the students realized what they were missing. Milton and I undertook to do something to fill the gap. We organized a student-run seminar, worked out a list of topics, assigned students to prepare papers, and guided the presentation and discussion. The other students benefitted and so did we. We were having our first teaching experience. For the moment, however, it helped keep my mind running in the grooves of my Harvard training

My friendship with Milton was solidified when a Columbia classmate invited us to join him in a long holiday in his family’s fishing camp on the French River in Northern Ontario, still a wild and unsettled area. It turned out, however, that our friend was ordered to work in his family’s business concern for the summer. We were invited to use the camp ourselves, and we did. So we spent a wonderful six weeks together. We drove north in my Model A Ford roadster until we reached a tiny settlement on the French River called Bon Air. There we parked the car at a general store where we hired some cots, some cooking utensils, a gasoline cookstove, and a canoe, and where we bought some canned and packaged foods as well as eggs and Canadian back bacon. The general store owner piled all these objects in his motorboat and, with the canoe in tow, took us out to our camp 3½ miles down the river on a tiny island in the stream. We were the only inhabitants. There he literally threw our stuff on the shore and took his leave. From now on, we had to depend on our canoe to get back and renew supplies at Bon Air.

Neither of us at first knew anything about canoeing, but we had good teachers by example in the Indians from a reservation across the river. Watching them, we soon learned the J stroke and became fairly competent. We canoed to Bon Air twice weekly and soon organized our camp. We had a privy some 50 yards away. We had the usual first experience trying to cook rice, but we learned to get along. We swam twice a day, and, as we gained confidence in the canoe, took overnight canoe trips down the river. These were fun, especially because of occasional rapids which we could run going down the river but had to portage around on the way back. The one thing we did not try was fishing. In fact, we became known along the river as those strange boys who did not fish, so many men returning in the late afternoon would throw us a fish or two. We had a valuable supplement to our diet of canned goods.

The thing we did do all day long, every day, was talk—about everything, but mostly economics. Milton was much less ideological then than he later became, so he was a very pleasant and agreeable companion; that was especially important in 1934, in the depths of the Depression when Roosevelt’s New Deal was just taking shape, when it included so much that was controversial, and when the menace of Hitler was becoming clearly visible.

As things turned out, however, the most important thing for me in that academic year of 1933-34 was the advent of Carrie [whom he would marry]. But that belongs in a chapter of its own.

…When I finished my graduate course work in 1935, I was given an instructorship at Harvard, I owed it to the sponsorship of Ed Mason, my old tutor. With all this arranged, we determined to get married. I was to have a first year to get started at Harvard, and Carrie was to have a year to complete her Columbia course. We would marry in June 1937. We told our parents and friends. Everyone was pleased.

…You will recall that on completing my graduate work at Columbia, I returned to Harvard as an instructor and tutor in 1936. I spent the first year on my own; then, following our marriage, Carrie joined me there. We lived in a comfortable little apartment at 31 Concord Avenue, near the RadcliffeYard.

It turned out to be an unsatisfactory time, which brought each of us into our only serious confrontations with discrimination. For Carrie it was a brush with what would now be called “sexism.” She heard that Wellesley was looking for a young instructor. She thought correctly that her graduate work and teaching experience qualified her. She appeared for an interview, which was conducted by John Dunlop, a Harvard professor. They reviewed her background, and, he conceded, she was qualified. And then he told her, with expressions of regret, that her application could go no further. Wellesley, a women’s college, wanted only a male.

My own problem was an example of that anti-Semitism that still infected Harvard and most other universities. During my time back at Harvard, I had taught Ec A and a course in Labor Market Economics, and I had tutored a full quota of economics majors in my tutorial rooms in Dunster House. I thought it had gone pretty well.

To this I should add the tale of an amusing development. When I returned to Cambridge in September 1937 together with Carrie, I was told by the department chairman that my salary, then $2,500 a year, would be raised by $200. And then he carefully explained that that was not because, as a married man, my expenses were higher. It was because I was married that he could add Radcliffe girls to my list of tutees. Needless to say, the relation of women to men has since changed radically. Harvard and Radcliffe are now fully merged. Women and men are now equally Harvard professors and Harvard students. The days when Radcliffe girls were thought to be at special and intolerable risk if they met an unmarried tutor have long gone.

In the spring of 1938, I received another summons from the chairman [Harold Burbank]. He received me cordially, and after the usual preliminary politenesses, he explained that it was time we discussed my future at Harvard. His opening was itself a warning about what was to come. “Now, Moe, we are both men of the world.” And then he went on to say that I had done well. I had a promising future. “But you must understand; we could not promote Jakey, so you must not expect to stay on here.” I had formed no such expectation, but I understood perfectly. “Jakey” was Jacob Viner, a truly notable economist. He had done brilliant theoretical work early. He was Taussig’s favorite student. Clearly, Harvard’s president at the time was a bar. He would not accept the appointment of Jews, something widely whispered. They might be scholars, but, by Lowell’s Boston Brahmin standards, they could not be gentlemen. So all this was hardly a complete surprise. But my chairman’s quiet but open expression of anti-Semitism was a shock.

I have often wondered whether it was not really a subtle way of ending my appointment without saying that I simply had not measured up. Perhaps, but that could hardly apply to Viner, who went on to do brilliant work, and who ended his career as a colleague of Einstein at the Institute for Advanced Study at Princeton. Had a Nobel Prize for Economics existed at the time, he would certainly have been a Nobel laureate.

So I left the interview knowing that I had to make plans to move. My opportunity was not long in coming. Later that same spring, I appeared again at Columbia for the defense of my dissertation, the last step on the way to the doctorate. The committee was chaired by Wesley Mitchell, the man whose course on business cycles I had dropped six year earlier. It made no difference to the examination. Apparently, I passed easily. Indeed my thesis won the Seligman Prize for the best of the year. When the committee adjourned, Mitchell asked me to stay behind. He wanted to ask me whether I would be willing to join the National Bureau to work with him on the Bureau’s business cycles project. My salary would be $3,500 year, a thousand dollars above my Harvard salary. In my circumstances it did not take me long to decide. In a couple of days he had my answer. I would be delighted. So now, after our first summer in Maine, Carrie and I moved to New York. I can guess now how the Bureau appointment had come about. My friend Milton Friedman (see Chapter Six), had just joined the Bureau with an appointment like my own, but to work on another subject. Milton was a friend and also the favorite student of Arthur F. Burns, at the time Mitchell’s chief assistant, who was already the really effective head of the business cycles work. My guess is that Milton became aware of Burns’s interest in finding an associate for business cycles to work especially on the cyclical role of inventories. My dissertation included a chapter on inventories. So he probably told Burns, and then events took their course.

 

Source:  Moses Abramovitz, Days Gone By: A Memoir for my Family (2001), pp. 32-34, 41-49, 77-79. (Link to download the memoir as .pdf)

_______________________

Stanford Faculty Memorial Resolution

MOSES ABRAMOVITZ
(1912-2000)

Moses Abramovitz, William Robertson Coe Professor of American Economic History Emeritus, died December 1, 2000, at Stanford University Hospital, just one month before reaching his eighty-ninth birthday.

Known by his family, friends, and colleagues as “Moe,” Abramovitz was one of the primary builders of Stanford’s Department of Economics. He taught at Stanford for almost thirty years, taking leave only during 1962-63 to work as economic advisor to the secretary general of the Organization for Economic Cooperation and Development in Paris. He served as chair from 1963 to 1965, and from 1971 to 1974, both critical junctures in the department’s history. During his tenure at Stanford and after his retirement in 1976, Moe gained international renown and admiration for his pioneering contributions to the study of long-term economic growth.

Moe was born in Brooklyn, New York, to a Romanian Jewish immigrant family. After graduating from Erasmus Hall High School, he entered Harvard in 1928. Like many of his generation, Moe’s interest in economics was stimulated by the experience of the Great Depression. So, in 1932 he continued his undergraduate studies of the subject at Columbia University, where he received his Ph.D. in 1939. At Columbia, Moe began a lifelong friendship with Milton Friedman. In later years, Moe liked to joke that he had been debating with Friedman for more than fifty years, and consistently winning — except when Milton was present. Columbia connections also led Moe to join the National Bureau of Economic Research in 1937, where he helped to launch the business cycle studies for which the Bureau became famous, working with such figures as Wesley Mitchell, Simon Kuznets and Arthur Burns.

Also at Columbia, Moe became re-acquainted with his Erasmus classmate Carrie Glasser, who was also working for her doctoral degree in economics. Moe and Carrie were married in June of 1937, and were devoted to each other until Carrie’s death in October 1999. When Moe came to Stanford in 1948, Carrie began what became a highly satisfying and successful career as a painter, sculptress and collage artist. Their only son, Joel, born in 1946, is a practicing neurosurgeon in Connecticut.

During World War II, Moe served first at the War Production Board, working with Simon Kuznets to analyze the limits of feasible production during wartime. He then moved to the Office of Strategic Services as chief of the European industry and trade section. During 1945 and 1946, he was economic advisor to the United States representative on the Allied Reparations Commission. Moe’s modest but strong character was well displayed in an episode during the postwar reparations debate. Treasury Secretary Henry Morgenthau had proposed a plan to deindustrialize the German economy. An OSS research team headed by Moe wrote a memorandum arguing that this plan would destroy Germany’s capacity to export, leaving it unable to pay for food and other essential imports. At a meeting with Moe and two other OSS economists, Ed Mason and Emile Despres, Morgenthau angrily asked: “Who is responsible for this?” Moe recalled: “Mason looked at Despres, and Emile looked at me. I had no one else to look at. The buck stopped with me. So, rather meekly, I said I was responsible.”

This anecdote and many others may be found in a charming memoir that Moe completed shortly before his death, “Days Gone By,” accessible on the Stanford Economics Department website.

At Stanford Moe began the studies of long-term economic growth that established his reputation among professional economists. A 1956 paper provided the first systematic estimates showing that forces raising the productivity of labor and capital were responsible for approximately half of the historical growth rate of real U.S. GDP, and close to three quarters of the growth rate of real GDP per capita. Subsequently he made seminal contributions in identifying the factors promoting and obstructing convergence in levels of productivity among advanced and developing countries of the world. For these studies and others, Moe received many academic honors. He was elected to the presidency of the American Economic Association (1979-80), the Western Economic Association (1988-89), and the Economic History Association (1992-93). From abroad came honorary doctorates from the University of Uppsala in Sweden (1985), and the University of Ancona in Italy (1992); he took special enjoyment from an invitation to become a fellow of the prestigious Academia Nazionale de Lincei in 1991 — “following Galileo with a lag,” he said, with a characteristic self-deprecatory twinkle.

Committee:

Paul A. David
Ronald McKinnon
Gavin Wright

Source: Stanford Report, July 9, 2003.

Image Source: Harvard Class of 1932, Twenty-fifth Anniversary Report (1957).

 

 

Categories
Courses Economic History Exam Questions Harvard

Harvard. Economics course descriptions, enrollments, final exams. 1915-16.

 

In this post I have assembled all the Harvard economics examinations I could find for the academic year 1915-16 and then supplement these with the annual enrollment data published in the President’s annual report which incidentally identifies the course instructors. Next I thought it would be even nicer to add course descriptions, but unfortunately I did not have access to the published 1915-16 announcement for the Division of History, Government, and Economics so I have added the course descriptions from 1914-15 or 1916-17 where the course titles and instructors exactly match.

For year-long courses, only the year-end final examination was included in the Harvard publication of examination papers, i.e. the mid-year final exams from January are missing for those courses. However, for the principles course and Taussig’s graduate theory course I have been able to find copies of those exams filed elsewhere in the Harvard archives (see notes).

Primarily for undergraduates:

Principles of Economics (Day with selected topics by Taussig)

For undergraduates and graduates
Statistics (Day)
Accounting (Davis)
European Industry and Commerce in the Nineteenth Century (Gay)
Economic and Financial History of the United States (Gay)
Money, Banking, and Commercial Crises (Anderson)
Economics of Transportation (Ripley)
Economics of Corporations (Ripley
Public Finance, including the Theory and Methods of Taxation (Bullock)
Trade Unionism and Allied Problems (Ripley)
Economic Theory (Taussig)
Principles of Sociology (Carver)
Economics of Agriculture (Carver)

Primarily for graduates
Economic Theory (Taussig)
The Distribution of Wealth (Carver)
Statistics: Theory, Method, and Practice (Day)
History and Literature of Economics to the year 1848 (Bullock)
Analytical Sociology (Anderson)
Public Finance (Bullock)

 

________________________

Principles of Economics (Day with selected topics by Taussig)

ECONOMICS A: Course announcement [1914-15]

[Economics] A. (formerly 1). Principles of Economics. Tu., Th., Sat., at 11.
Professor TAUSSIG and Asst. Professor DAY and five assistants.

Course gives a general introduction to economic study, and a general view of Economics for those who have not further time to give to the subject. It undertakes a consideration of the principles of production, distribution, exchange, money, banking, international trade, and taxation. The relations of labor and capital, the present organization of industry, and the recent currency legislation of the United States will be treated in outline.

The course will be conducted partly by lectures, partly by oral discussion in sections. A course of reading will be laid down, and weekly written exercises will test the work of students in following systematically and continuously the lectures and the prescribed reading. Course A may not be taken by Freshmen without the consent of the instructor.

ECONOMICS A: Enrollment [1915-16]

 [Economics] A. Asst. Professor Day; and Dr. J. S. Davis and Mr. P. G. Wright, Dr. Burbank, and Messrs. Monroe, Lincoln, R.E. Richter, and Van Sickle. With Lectures on selected topics by Professor Taussig. — Principles of Economics.

Total 477: 1 Graduate, 28 Seniors, 111 Juniors, 278 Sophomores, 13 Freshmen, 46 Other.

ECONOMICS A: Mid-Year Examination [1915-16]

Plan your answers carefully before writing. Write concisely. Arrange your answers strictly in the order of the questions, beginning each on a new page.

  1. What are the characteristic features of each of the following: (a) horizontal combination; (b) a bill of exchange; (c) bimetallism; (d) marginal cost; (e) subsidiary coinage?
  2. Give four important economic advantages of (a) the complex division of labor; (b) large-scale production; (c) the corporate form of organization.
  3. Indicate any important connections existing between (a) the corporation and large-scale production; (b) large-scale production and dumping; (c) dumping and a protective tariff; (d) a protective tariff and the geographical division of labor.
  4. What conditions of demand and supply tend to promote, what to impede, organized speculation? What are the functions, and what the chief consequences of, organized speculation in agricultural products?
  5. In what ways, if at all, is monopoly price affected by (a) cost of production per unit? (b) an elastic demand for the product? Illustrate by diagrams, assuming conditions of (1) constant cost, (2) decreasing cost.
  6. Briefly describe the Panic of 1907 in New York. What provisions of the Federal Reserve Act do you consider most likely to be effective in preventing or allaying future financial panics in the United States? Give your reasons in detail.
  7. What has been the general course of the sterling exchange rate since the beginning of 1914? What factors have been influential in causing changes in the rate? How has each factor operated?

Source note:  This mid-year examination was found at Harvard University Archives. Department of Economics, Course reading lists, syllabi, and exams 1913-1992. (UA V 349.295.6) Box 1, Folder “Economics I, Final Exams 1913-1939”.

ECONOMICS A: Final Examination [1915-16]

Plan your answers carefully before writing. Write concisely. Arrange your answers strictly in the order of the questions, beginning each on a new page.

  1. What is meant by (a) marginal cost; (b) the representative firm? How, if at all, is marginal cost connected with the short- and long-time values of (a)fresh vegetables; (b) wheat; (c)a railroad rate; (d) a gold dollar?
  2. Explain: (a) free coinage; (b) undervalued metal; (c) overissue; (d) “creation of deposits”; (e) bank reserve; (f) currency premium.
  3. “Think of it! British ships are bringing in foreign tires; British money is going abroad to pay for them1; and British motorists are using them. The available supplies of British-made tires are ample for all needs. Imported tires are inessentials; they hurt British credit2, they lower the exchange of the English pound3, they increase freights4, they make necessities dearer5, and increase our national indebtedness6.” To what extent is the reasoning valid at the several points indicated?
  4. Explain what is meant by (a) the unearned increment of land; (b) “the unearned increment of railways”; (c) increment taxes; (d)the incidence of taxes on land; (e) the Single Tax.
  5. What effects upon wages, if any, should you expect to result from (a) free industrial education; (b) collective bargaining; (c) limitation of output by organized labor; (d) introduction of labor-saving machinery?
  6. What should you expect to be the effect of immigration into the United States on (a) the increase of population here; (b) wages in the United States; (c) American urban rents; (d) profits of American business men?
  7. What is to be said for and against (a) unemployment insurance; (b) compulsory arbitration for public service industries; (c) profit-sharing as an agency for industrial peace?
  8. Explain: (a) restraint of trade at common law; (b) restraint of trade under United States statute law; (c) “rule of reason”; (d) “unfair competition”; (e) Kartel.

 

________________________

Statistics (Day)

ECONOMICS 1a1: Course announcement [1914-15]

[Economics] 1a 1hf. Statistics. Half-course (first half-year). Mon., Wed., Fri., at 11. Asst. Professor DAY, assisted by Mr. F. E. RICHTER.

This course will deal primarily with the elements of statistical method. The following subjects will be considered: methods of collecting and tabulating data; the construction and use of diagrams; the use and value of the various types and averages; index-numbers; dispersion; interpolation; correlation. Special attention will be given to the accuracy of statistical material. In the course of this study of statistical method, examples of the best statistical information will be presented, and the best sources will be indicated. Population and vital statistics will be examined in some measure, but economic statistics will predominate.

Laboratory work in the solution of problems and the preparation of charts and diagrams will be required.

ECONOMICS 1a1: Enrollment [1915-16]

[Economics] 11hf. Asst. Professor Day, assisted by Mr. Cox. — Statistics.

Total 44: 2 Graduates, 17 Seniors, 18 Juniors, 7 Other.

 

ECONOMICS 1a1: Final Examination [1915-16]

  1. What is meant by “the statistical method”? What is the scientific importance of the method? What are its limitations?
  2. Describe concisely the essential steps in the preparation for a population census.
  3. Sketch briefly the history of wage statistics in the United States.
  4. Describe in detail, and criticize, the Babson method of forecasting business conditions.
  5. Explain briefly: (a) law of statistical regularity; (b) probable error; (c) series; (d) mode; (e) the normal frequency curve; (f) skewness.
  6. Formulate a set of rules for the construction of frequency tables and graphs.
  7. By what different statistical devices may the structure — or distribution — of two different groups of data be compared?
  8. Explain briefly: correlation; ratio of variation.
    Criticise fully the following statement: “A very large degree of regression — that is, a large deviation of the line of regression from the line of equal proportional variation — indicates a slight degree of correlation.”

 

________________________

Accounting (Davis)

ECONOMICS 1b2: Course Announcement [1914-15]

[Economics] 1b 2hf. Accounting. Half-course (second half-year). Lectures, Mon., Wed., and (at the pleasure of the instructor) Fri., at 1.30; problems and laboratory practice, two hours a week. Dr. J. S. Davis, assisted by Mr. F. E. RICHTER and—.

This course will deal with the construction and the interpretation of accounts of various types of business units, designed to show the financial status at a particular time, the financial results obtained during a period of time, and the relation between the results and the contributing factors. In other words, it will be concerned with the measurement, in terms of value, of economic instruments, forces, products, and surpluses.

Some attention will necessarily be given to the fundamentals of book-keeping, but emphasis will be placed chiefly upon the accounting principles underlying valuation and the determination of profits and costs. Problem work will be regularly assigned, and published reports of corporations will serve as material for laboratory work.

ECONOMICS 1b2: Enrollment [1915-16]

[Economics] 1bhf. Dr. J. S. Davis, assisted by Mr. Cox. — Accounting.

Total 116: 49 Seniors, 62 Juniors, 3 Sophomores, 2 Other.

 

ECONOMICS 1b2: Final Examination [1915-16]

Be concise. Reserve at least 45 minutes for Question 8. If desired, one of the first five questions may be omitted.

  1. What purposes are served by a Journal? a Ledger? Is it possible to keep complete and accurate accounts with these books alone?
    b. Name five other account books commonly found, and indicate briefly the nature and special function of each.
  2. Explain briefly: posting, contingent liability, corporate surplus, amortization table, secret reserve.
  3. With respect to each of the following, indicate (preferably in tabular form) (a) whether it would normally show a debit or credit balance, (b) whether it would appear on balance sheet or income statement, and (c) what kindof account it represents.

Rentals of Properties Owned
Sinking Fund Securities
Insurance Unexpired
Reserve for Accrued Depreciation
Depreciation on Equipment
Premium on Stock Issued
Advances to Subsidiary Companies
Extraordinary Flood Damages

  1. Draft journal entries (omiting explanations) for the following transactions of the General Utility Company:
    1. Sale of six desks to Jackson & Jackson, @ $15, 30 days, receiving in part payment their 30-day note for $50.
    2. Declaring dividends of $200,000, setting aside out of current income a fire insurance reserve of $100,000, and adding the balance of the year’s income ($60,000) to the surplus.
    3. Making the semi-annual interest payment on a million-dollar 6 per cent bond issue, the bond premium being simultaneously amortised to the extent of $2000.
    4. Loss by fire of a building which cost $60,000, and upon which depreciation of $10,000 had accrued and been allowed for.
  2. What is the purpose of a balance sheet? What are its essential elements? What are the main items or groups of items on the balance sheet of a railroad company? At what points are balance sheets frequently defective, inaccurate, or misleading?
  3. Do the following, in a railroad report, ordinarily signify improvement or retrogression? Under what circumstances, if any, might each signify the opposite? How could you ascertain which was actually signified?
    1. Decline in operating ratio.
    2. Increase in maintenance of freight cars per freight car.
    3. Decrease in freight train miles.
  4. Explain the purpose of the “funding accounts peculiar” to governmental accounting, and illustrate their use.
    b. What accounting distinctions are of especial importance in municipal accounting?
  5. Below are comparative figures (in thousands of dollars) of a company manufacturing railway equipment. Summarize what they reveal of its history, condition, and policy, commending or criticising the statements or policy as occasion requires.

 

Income Account, Years ended December 31
1907 1908 1909 1910 1911 1912 1913
Gross Earnings Not reported 5,920 7,843 10,035 6,160 9,041 7,688
Operating and Mfg. Expenses, etc. 4,775 5,782 7,734 4,793 6,600 6,216
Depreciation and Maintenance 170 194 350 150 360 *
Net Earnings 2,320 975 1,866 1,951 1,217 2,081 1,472
Bond Interest 217 209 203 196 232 357 350
Dividends 1,485 1,350 945 945 945 945 945
Surplus for the Year 618 **584 718 810 40 779 177

*Included in “operating expenses.”  **Deficit.

 

General Balance Sheet, December 31
Assets 1907 1908 1909 1910 1911 1912 1913
Plants, Properties, etc. 30,291 30,536 30,568 30,267 33,746 33,373 33,320
Inventories 2,341 1,914 1,927 2,210 1,622 1,927 1,593
Stocks, Bonds, etc. 185 217 222 242 400 704 686
Accounts Receivable 2,349 1,212 1,667 1,464 1,148 1,986 1,411
Other Items 84 75 38 32 28 41 48
Cash 264 344 382 871 1,484 1,225 1,814
Total 35,514 34,298 34,804 35,086 38,428 39,256 38,872
Liabilities
Common Stock 13,500 13,500 13,500 13,500 13,500 13,500 13,500
Preferred Stock (7% cumulative) 13,500 13,500 13,500 13,500 13,500 13,500 13,500
Bonded Debt 4,223 4,083 3,945 3,808 7,172 7,037 6,901
Accounts Payable 1,239 588 672 212 148 350 186
Bills Payable 50 200
Reserves for Dividends, Interest, Taxes, etc. 147 156 197 266 268 251 260
Surplus 2,855 2,271 2,990 3,800 3,840 4,618 4,525
Total 35,514 34,298 34,804 35,086 38,428 39,256 38,872

 

________________________

European Industry and Commerce in the Nineteenth Century (Gay)

ECONOMICS 2a1: Course announcement [1914-15]

[Economics] 2a1hf. European Industry and Commerce in the Nineteenth Century. Half-course(first half-year).Tu., Th., Sat., at 9. Professor GAY, assisted by—.

Course 2undertakes to present the general outlines of the economic history of western Europe since the Industrial Revolution. Such topics as the following will be discussed: the economic aspects of the French Revolution and the Napoleonic régime, the Stein-Hardenberg reforms, the Zoll-Verein, Cobden and free trade in England, labor legislation and social reform, nationalism and the recrudescence of protectionism, railways and waterways, the effects of transoceanic competition, the rise of industrial Germany.

Since attention will be directed in this course to those phases of the subject which are related to the economic history of the United States, it may be taken usefully before Economics 2b.

ECONOMICS 2a1: Enrollment [1915-16]

[Economics] 2a1hf. Professor Gay, assisted by Messrs. A. H. Cole and Ryder.— European Industry and Commerce in the Nineteenth Century.

Total 94: 23 Graduates, 17 Seniors, 33 Juniors, 16 Sophomores, 5 Other.

 

ECONOMICS 2a1: Final Examination [1915-16]

  1. Speaking of the industrial revolution in England, a writer says: “It is to a revolution in three industries, — agriculture, cotton and iron, — that this transformation is principally due.” Do you agree? Give your reasons.
  2. Account historically for the present condition of the agricultural laborer in England, in East Prussia. What have been the social consequences in both cases?
  3. Hadley says of railway construction: “The Englishman built for the present and future both; the American chiefly for the future.” Account for this difference, and show its effect on capitalization, on service and on inter-railway relations.
  4. Trace the influence of the agrarian and industrial interests on tariff legislation in Germany and France since 1880.
  5. Give an account of the development of the iron and steel industry in England and Germany in the last half of the nineteenth century. Account for the later development in the latter country, and trace the competition between the Ruhr and Lorraine districts.

(Take one of the following two questions)

  1. Comment on Ashley’s statement regarding English exports:

“We shall more and more exhaust our resources of coal, and we shall devote ourselves more and more to those industries which flourish on cheap labor.”

  1. How have the laboring people of England by voluntary collective action tried to meet the exigencies of the modern industrial system? Compare with Germany.

 

________________________

Economic and Financial History of the United States (Gay)

ECONOMICS 2b2: Course Announcement [1914-15]

[Economics] 22hf. Economic and Financial History of the United States. Half-course (second half-year). Tu., Th., Sat., at 9. Professor GAY, assisted by —.

The following are among the subjects considered: aspects of the Revolution and commercial relations during the Confederation and the European wars; the history of the protective tariff policy and the growth of manufacturing industries; the settlement of the West and the history of transportation, including the early canal and turnpike enterprises of the states, the various phases of railway building and the establishment of public regulation of railways; banking and currency experiences; various aspects of agrarian history, such as the public land policy, the growth of foreign demand for American produce and the subsequent competition of other sources of supply; certain social topics, such as slavery and its economic basis, and the effects of immigration.

ECONOMICS 2b2: Enrollment [1915-16]

[Economics] 22hf. Professor Gay, assisted by Messrs. A. H. Cole and Ryder. — Economic and Financial History of the United States.

Total 94: 23 Graduates, 17 Seniors, 33 Juniors, 16 Sophomores, 5 Other.

 

ECONOMICS 2b2: Final Examination [1915-16]

  1. “The expulsion of the French from Canada made it possible (for the American colonies) to dispense with English protection. The commercial restrictions made it to their interest to do so.” Do you agree? Give your reasons for or against.
  2. “As to the strength of slavery as an institution in Southern society after it had been thoroughly established, its basis was partly economic and partly social.” Explain. Which do you think the more fundamental? Why?
  3. (a) Give the reasons for the turn in our favor of the balance of trade in the seventies. (b) Into what periods would you divide the history of our export trade since that time? Characterize each period. What do you think are the probabilities for the future? Give your reasons.
  4. Compare the marketing of grain with the marketing of wool. Why the difference?
  5. In how far were the policies of the national government responsible for the panics of 1837 and 1893? Give your reasons.
  6. (a) Describe briefly the development of the iron industry in the United States. (b) What effect has this development had upon American shipping before and after 1870?

The following questions are for graduates who did not take the tests:

  1. Take one of the following subjects: (a) the history of American agriculture since 1860; or (b) Manufacturing development in the United States before 1860; or (c) the history of American transportation since 1860. Outline the periods and topics you would discuss in lecturing on it. Give also a short list of the chief books or papers you would consult, with critical estimates.
  2. What criteria would you hold most significant in determining the successful application of protection to young industries. Draw your evidence from the manufactures we have considered.

 

________________________

Money, Banking, and Commercial Crises (Anderson)

ECONOMICS 3: Course Announcement [1914-15]

[Economics] 3. Money, Banking, and Commercial Crises. Mon., Wed., Fri., at 2.30. Asst. Professor ANDERSON, assisted by —.

This course undertakes a theoretical, descriptive, and historical study of the main problems of money and banking. Historical and descriptive materials, drawn from the principal systems of the world, will be extensively used, but will be selected primarily with reference to their significance in the development of principles, and with reference to contemporary practical problems. Foreign exchange will be studied in detail. Attention will be given to those problems of money and credit which appear
most prominently in connection with economic crises. Though emphasis will be thrown upon the financial aspects of crises, the investigation will cover also the more fundamental factors causing commercial and industrial cycles.

ECONOMICS 3: Enrollment [1915-16]

[Economics] 3. Asst. Professor Anderson, assisted by Mr. Silberling. — Money, Banking and Commercial Crises.

Total 69: 2 Graduates, 25 Seniors, 31 Juniors, 3 Sophomores, 8 Other.

 

ECONOMICS 3: Final Examination [1915-16]

Omit either question 6 or 7.

  1. State and discuss Fisher’s version of the quantity theory of money.
  2. Discuss the relations of the banks and the stock exchange.
  3. Contrast the Bank of England with the Banque de France:
    (a) with reference to reserves;
    (b) with reference to the discount rate;
    (c) with reference to specie payments;
    (d) with reference to relations with the government;
    (e) with reference to foreign exchange policy.
  4. In precisely what ways does our Federal Reserve system seek to remedy the defects in our banking system?
  5. Discuss the development of State banking since the Civil War. Compare it with the development of the National Bank system. Explain the tendencies.
  6. Give an account of the main movements in the prices of the war stocks since Oct. 1, 1915, and explain these movements as far as you can: (a) by reference to general causes; (b) by reference to factors affecting particular securities as far as you know them.
  7. Explain the movements in demand sterling since the outbreak of the War. Give figures and dates as accurately as you can.
  8. Summarize Wesley Mitchell’s theory of business cycles.
  9. For what purposes does the farmer need credit? What is the extent of agricultural indebtedness in different sections in the United States? What agencies supply credit to the farmer? What rates of interest does the farmer pay in different parts of the country?
  10. Contrast the Panic of 1893 with the Panic of 1914.

 

________________________

Economics of Transportation (Ripley)

ECONOMICS 4a1: Course Announcement [1914-15]

[Economics] 41hf. Economics of Transportation. Half-course (first half-year). Tu., Th., Sat., at 11. Professor RIPLEY, assisted by —.

A brief outline of the historical development of rail and water transportation in the United States will be followed by a description of the condition of transportation systems at the present time. The four main subdivisions of rates and rate-making, finance, traffic operation, and legislation will be considered in turn. The first deals with the relation of the railroad to shippers, comprehending an analysis of the theory and practice of rate-making. An outline will be given of the nature of railroad securities, the principles of capitalization, and the interpretation of railroad accounts. Railroad operation will deal with the practical problems of the traffic department, such as the collection and interpretation of statistics of operation, pro-rating, the apportionment of cost, depreciation and maintenance, etc. Under legislation, the course of state regulation and control in the United States and Europe will be traced.

ECONOMICS 4a1: Enrollment [1915-16]

[Economics] 4a 1hf. Professor Ripley, assisted by Mr. Cameron. — Economics of Transportation.

Total 121: 3 Graduates, 47 Seniors, 54 Juniors, 7 Sophomores, 10 Other.

ECONOMICS 4a1: Final Examination [1915-16]

  1. Discuss the propriety of the capitalization by a railroad of a surplus which had gradually accumulated during a period of twenty or more years. Would the recency of the surplus make any difference? How about the geographical location of the road?
  2. Describe the existing situation as concerns the relation of American railroads to their employees.
  3. What are the prime essentials of a railroad reorganization, necessary to insure its success?
  4. In case of the creation of a Congressional commission on railway legislature, what are the topics which it would probably consider?
  5. Outline the means which have been employed to bring about unity of action among the hard coal roads as to prices.
  6. State briefly for the leading countries which have taken over their railways as government enterprises, the peculiar circumstances which have no counterpart in the American situation.
  7. What is the trouble with the so-called basing point system?
  8. What is the present condition of affairs concerning the relation of railroads to water lines, coastwise or lake?
  9. When and how did the conflict of Federal and state powers over regulation of common carriers first become acute?
  10. Why was the United States Commerce Court ‘abolished’ judging by the tenor of its decisions?

 

________________________

Economics of Corporations (Ripley)

ECONOMICS 4b2: Course Announcement [1914-15]

[Economics] 42hf. Economics of Corporations. Half-course (second half-year). Tu., Th., Sat., at 11. Professor RIPLEY, assisted by —.

This course will treat of the fiscal and industrial organization of capital, especially in the corporate form. The principal topic considered will be industrial combination and the so-called trust problem. This will be broadly discussed, with comparative study of conditions in the United States and Europe. The development of corporate enterprise, promotion, and financing, accounting, liability of directors and underwriters, will be described, not in their legal but in their economic aspects; and the effects of industrial combination upon efficiency, profits, wages, prices, the development of export trade, and international competition will be considered in turn.

ECONOMICS 4b2: Enrollment [1915-16]

[Economics] 4hf. Professor Ripley, assisted by Mr. Cameron. — Economics of Corporations.

Total 115: 9 Graduates, 39 Seniors, 49 Juniors, 9 Sophomores, 1 Freshman, 8 Other.

 

ECONOMICS 4b2: Final Examination [1915-16]

  1. Discuss critically the “economics of Industrial Combination.”
  2. What peculiarity of the American situation has given especial prominence to the holding company, in contrast with European countries?
  3. What principle of corporate finance, not of commercial practice, is illustrated by the experience of the following companies? Limit each answer to five words.
    1. U.S. Leather Co.
    2. International Mercantile Marine Co.
    3. American Ice Co.
    4. U.S. Steel Corporation.
    5. American Tobacco Co.
    6. The Glucose combination.
    7. The Asphalt combination.
  4. What is the most insistent feature in an industrial reorganization? How is the desired result commonly brought about?
  5. Outline the relation of organized labor to the amendment of the Sherman Act in 1914.
  6. “Competitors must not be oppressed or coerced. Fraudulent or unfair, or oppressive rivalry must not be pursued….Then, too, prices must not be arbitrarily fixed or maintained … an artificial scarcity must not be produced….The public is also injured if quality be impaired….Other injuries are done, if the wages of the laborer be arbitrarily reduced, and if the price of raw material be artificially depressed.”
    Associate each of the foregoing practices named in a recent judicial opinion with some particular industrial combination.
  7. How successful has the Department of Justice been in effecting the corporate dissolution of combinations? Outline the experience.
  8. Describe those factors of British corporate financial practise which are essentially different from our own.
  9. Compare the organization of the American and German combinations in the iron and steel industries; briefly, point by point.
  10. If high prices constitute a grievance of the public against industrial combination, what are the objections to an attempt to regulate these prices directly by law? Discuss the proposition from as many points of view as possible.

 

________________________

Public Finance, including the Theory and Methods of Taxation (Bullock)

ECONOMICS 5: Course Announcement [1914-15]

[Economics] 5. Public Finance, including the Theory and Methods of Taxation. Mon., Wed., Fri., at 9. Professor BULLOCK.

This course covers the entire field of public finance, but emphasizes the subject of taxation. After a brief survey of the history of finance, attention is given to public expenditures, commercial revenues, administrative revenues, and taxation, with consideration both of theory and of the practice of various countries. Public credit is then studied, and financial legislation and administration are briefly treated.

Systematic reading is prescribed, and most of the exercises are conducted by the method of informal discussion. Candidates for distinction will be given an opportunity to write theses.

Graduate students are advised to elect Economics 31.

ECONOMICS 5: Enrollment [1915-16]

[Economics] 5. Professor Bullock. — Public Finance, including the Theory and Methods of Taxation.

Total 60: 27 Seniors, 28 Juniors, 5 Other.

 

ECONOMICS 5: Final Examination [1915-16]

  1. Trace historically the position occupied by the customs revenue in the finances of the United States. What principles should be observed in establishing a system of customs duties? Discuss the incidence of these duties.
  2. To what extent and for what reasons has the working of the general property tax in Switzerland been different from the working of the same tax in the United States?
  3. Discuss briefly and concisely the characteristic features of three of the following: (a) The impôt-personnel mobilier; (b) The French business tax; (c) The Prussian business tax; (d) inheritance taxes in the United States.
  4. Explain and discuss critically the methods employed in the taxation of incomes in England and in Prussia.
  5. (a) What are the different theories regarding the best method of apportioning taxes?
    (b) Distinguish between “funded” and “unfunded” incomes. On what grounds can the heavier taxation of funded incomes be urged?
  6. What principles should govern the prices charged for the services of public commercial undertakings?
  7. Enumerate and discuss critically all the maxims, or canons, of taxation, with which you are familiar.
  8. State either the case for or the case against the single tax.

 

________________________

Trade Unionism and Allied Problems (Ripley)

ECONOMICS 6a1: Course Announcement [1914-15]

[Economics] 61hf. Trade Unionism and Allied Problems. Half-course (first half-year). Tu., Th., Sat., at 10. Professor RIPLEY, assisted by —.

This course will deal mainly with the economic and social relations of employer and employed. Among the topics included will be: the history of unionism; the policies of trade unions respecting wages, machinery, output, etc.; collective bargaining; strikes; employers’ liability and workmen’s compensation; efficiency management; unemployment, etc., in the relation to unionism, will be considered.

Each student will make at least one report upon a labor union or an important strike, from the original documents. Two lectures a week, with one recitation, will be the usual practice.

ECONOMICS 6a1: Enrollment [1915-16]

[Economics] 6a hf. Professor Ripley, assisted by Mr. Weisman. — Trade-Unionism and Allied Problems.

Total 61: 24 Seniors, 29 Juniors, 1 Sophomore, 7 Other.

 

ECONOMICS 6a1: Final Examination [1915-16]

  1. Illustrate by a sketch the interrelation between the constituent parts of the American Federation of Labor.
  2. Criticise the following premium wage plans for mounting “gem” electric lamp bulbs.
Daily Output Wage per thousand
Under 900 $1.03
900-1000 $1.07
1000-1100 $1.12
Over 1100 $1.17
  1. Have you any impression whether Webb favors craft or industrial unionism? What instances does he cite?
  2. Define (a) Federal union; (b) Device of the Common Rule? (c) Jurisdiction dispute.
  3. Is there any real difference between an “irritation strike ” of the I. W. W.and the British “strike in detail”?
  4. Contrast the British and American policies of trade union finance, showing causes and results.
  5. Describe the Hart, Schaffner and Marx plan of dealing with its employees.
  6. Is the Standard Wage merely the minimum for a given trade or not? Discuss the contention that it penalizes enterprise or ability.
  7. Is there any relation logically between the attitude of labor toward piece work and the relative utilization of machinery?
  8. What is the nature of the business transacted at the annual convention of the American Federation of Labor?

 

________________________

Economic Theory (Taussig)

ECONOMICS 7a1: Course Announcement [1916-17]

[Economics] 7ahf. Economic Theory. Half-course(first half-year). Tu., Th., at 2.30, and (at the pleasure of the instructor) Sat., at 11. Professor TAUSSIG.

Course 7a undertakes a survey of economic thought from Adam Smith to the present time. Considerable parts of the Wealth of Nations and of J. S. Mill’s Principles of Political Economy will be read, as well as selected passages from the writings of contemporary economists. No theses or other set written work will be required. The course will be conducted chiefly by discussion. It forms an advantageous introduction to Economics 7b.

Students who have attained in Economics a grade sufficient for distinction (or B) are admitted without further inquiry. Others must secure the consent of the instructor.

ECONOMICS 7a1: Enrollment [1915-16]

[Economics] 7a 1hf. Professor Taussig. — Economic Theory.

Total 27: 12 Graduates, 5 Seniors, 5 Juniors, 1 Sophomore, 4 Other.

 

ECONOMICS 7a1: Final Examination [1915-16]

Arrange your answers strictly in the order of the questions.

  1. “The wages of the inferior classes of workmen, I have endeavored to show in the first book, are everywhere necessarily regulated by two different circumstances: the demand for labor, and the ordinary or average price for provisions. The demand for labor, according as it happens to be either increasing, stationary, or declining, or to require an increasing, stationary or declining population, regulates the subsistence of the laborer and determines in what degree it shall be either liberal, moderate, or scanty.”
    Explain (1) in what way Adam Smith analyzed the “demand for labor”; (2) the nature of the reasoning which led to his conclusions regarding the influence on wages of increasing or declining national wealth.
  2. Explain in what way J. S. Mill analyzed the demand for labor, and wherein his analysis resembled Adam Smith’s, wherein it differed; and consider whether Mill’s conclusions regarding the influence of increasing national wealth on wages were similar to Adam Smith’s.
  3. Explain:
    (a) The Physiocratic notion concerning productive labor;
    (b) Adam Smith’s distinction between productive and unproductive labor;
    (c) Adam Smith’s doctrine as to the way in which equal capitals employed in agriculture, in manufactures, in wholesale or retail trade, put in motion different quantities of productive labor.
    What reasoning led Adam Smith to arrange industries in the order of productiveness indicated in (c) and what have you to say in comment on it
  4. Why, according to Adam Smith, is there rent from land used for growing grain? from land used for pasture? from mines? What would a writer like Mill say of these doctrines of Adam Smith’s?
  5. How does Mill (following Chalmers) explain the rapid recovery of countries devastated by war? Do you think the explanation sound?
  6. Wherein is Mill’s analysis of the causes of differences in wages similar to Adam Smith’s, wherein different?
  7. What, according to Mill, is the foundation of private property? What corollaries does he draw as regards inheritance and bequest? What is your instructor’s view on the justification of inheritance and bequest?
  8. Explain wherein there are or are not ” uman costs” in the savings of the rich, of the middle classes, and of the poor; and wherein there are or are not “economic costs” in these several savings.
  9. Hobson says: (a) that” the traditional habits of ostentatious waste and conspicuous leisure . . . induce futile extravagance in expenditure”; (b) that “the very type of this expenditure is a display of fireworks; futility is of its essence”; (c) that “the glory of the successful sportsman is due to the fact that his deeds are futile. And this conspicuous futility is at the root of the matter. The fact that he can give time, energy, and money to sport testifies to his possession of independent means.” Consider what is meant by “futility” in these passages; and give your own opinion on the significance of “sport.”
  10. Explain the grounds on which Hobson finds little promise for the future in (a) consumers’ cooperation; (b) producers’ cooperation; (c) syndicalism.

 

________________________

Principles of Sociology (Carver)

ECONOMICS 8: Course Announcement [1916-1917]

[Economics] 8. Principles of Sociology. Mon., Wed., Fri., at 9. Professor CARVER, assisted by Mr. —.

A study in social adaptation, both passive and active. Problems of race improvement, moral adjustment, industrial organization, and social control are considered in detail.  [Note: in 1916-17 this became a two-term course]

ECONOMICS 81: Enrollment [1915-16]

[Economics] 8 1hf. Professor Carver, assisted by Mr. Bovingdon.— Principles of Sociology.

Total 130: 14 Graduates, 51 Seniors, 45 Juniors, 5 Sophomores, 15 Other.

 

ECONOMICS 81: Final Examination [1915-16]

  1. How would you distinguish between progress and change?
  2. Just what is meant by self-centered appreciation? Should the range of the average individual’s appreciations be widened? Give reasons for your answer.
  3. What do you think of the economic test of the individual’s fitness for survival?
  4. What is the function of religion? To what extent do you think that it is performing its function in the United States?
  5. What is the function of an educational institution? To what extent do you think that Harvard University is performing its function?
  6. What effect do you think that the increase of government ownership and operation of industrial capital in the United States will have upon the “open road to talent”?

 

________________________

Economics of Agriculture (Carver)

ECONOMICS 91: Course Announcement [1914-15]

[Economics] 9 1hf. Economics of Agriculture. Half-course (first half-year). Mon., Wed., and (at the pleasure of the instructor) Fri., at 10. Professor CARVER.

A study of the relation of agriculture to the whole industrial system, the relative importance of rural and urban economics, the conditions of rural life in different parts of the United States, the forms of land tenure and methods of rent payment, the comparative merits of large and small holdings, the status and wages of farm labor, the influence of farm machinery, farmers’ organizations, the marketing and distribution of farm products, agricultural credit, the policy of the government toward agriculture, and the probable future of American agriculture.

ECONOMICS 91: Enrollment [1915-16]

[Economics] 9 1hf. Professor Carver, assisted by Mr. Shaulis.— Economics of Agriculture.

Total 58: 4 Graduates, 32 Seniors, 16 Juniors, 3 Sophomores, 3 Other.

 

ECONOMICS 91: Final Examination [1915-16]

  1. What are the factors which determine the migration of rural people; of urban people?
  2. What are the chief periods in the development of American Agriculture, and how would you characterize each period?
  3. In what ways could a citizen acquire title to a piece of the public land of the United States at the following dates, 1850, 1870, 1900?
  4. What do you regard as the necessary steps to the solution of the problem of rural credit in the United States? Explain your reasons.
  5. What are the essentials to be achieved in the building up of a market for agricultural products?
  6. Discuss the place of animal husbandry in the economy of the farm and also in the economy of food production from the standpoint of society in general.
  7. Summarize the effects of modern farm machinery. Discuss the degree of its utilization in different sections of the United States.
  8. Outline briefly a scheme for the organization of a rural community, and give your reasons for the main features of your scheme.
  9. Outline the chief areas of production in the United States of the following crops: Potatoes, wheat, oats, hay and forage.
  10. What are the chief forms of tenancy in the United States, and where is each form most common?

 

________________________

Economic Theory (Taussig)

ECONOMICS 11: Course Announcement [1914-15]

[Economics] 11. Economic Theory. Mon., Wed., Fri., at 2.30. Professor TAUSSIG.

Course 11 is intended to acquaint the student with some of the later developments of economic thought, and at the same time to train him in the critical consideration of economic principles and the analysis of economic conditions. The exercises are accordingly conducted mainly by the discussion of selected passages from the leading writers; and in this discussion the students are expected to take an active part. The writings of J. S. Mill, Cairnes, F. A. Walker, Clark, Marshall, Böhm-Bawerk, and other recent authors, will be taken up. Attention will be given chiefly to the theory of exchange and distribution.

ECONOMICS 11: Enrollment [1915-16]

[Economics] 11. Professor Taussig. — Economic Theory.

Total 29: 18 Graduates, 1 Grad.Bus., 6 Seniors, 3 Radcliffe, 1 Other.

 

ECONOMICS 11: Mid-year Examination [1915-16]

Arrange your answers strictly in the order of the questions.

  1. On what grounds is it contended that there is a circle in Walker’s reasoning on the relation between wages and business profits? What is your opinion on this rejoinder: that Walker, in speaking of the causes determining wages, has in mind the general rate of wages, whereas in speaking of profits he has in mind the wages of a particular grade of labor?
  2. According to Ricardo, neither profits of capital nor rent of land are contained in the price of exchangeable commodities, but labor only.” — Thünen.
    Is there justification for this interpretation of Ricardo?
  3. “Instead of saying that profits depend on wages, let us say (what Ricardo really meant) that they depend on the cost of labour. . . . The cost of labour is, in the language of mathematics, a function of three variables: the efficiency of labor; the wages of labour (meaning thereby the real reward of the labourer); and the greater or less cost at which the articles composing that real reward can be produced or procured.”   — J. S. Mill.
    Is this what Ricardo really meant? Why the different form of statement by Mill? What comment have you to make on Mill’s statement?
  4. State resemblances and differences in the methods of analysis, and in the conclusions reached, between (a) the temporary equilibrium of supply and demand (e.g. in a grain market), as explained by Marshall; (b) “two-sided competition,” as explained by Böhm-Bawerk; (c) equilibrium under barter, as explained by Marshall.
  5. Explain concisely what is meant in the Austrian terminology by “value,” “subjective value,” “subjective exchange value,” “objective exchange value.”
    Does the introduction of “subjective exchange value” into the analysis of two-sided competition lead to reasoning in a circle?
  6. “Suppose a poor man receives every day two pieces of bread, while one is enough to allay the pangs of positive hunger, what value will one of the two pieces of bread have for him? The answer is easy enough. If he gives away the piece of bread, he will lose, and if he keeps it he will secure, provision for that degree of want which makes itself felt whenever positive hunger has been allayed. We may call this the second degree of utility. One of two entirely similar goods is, therefore, equal in value to the second degree in the scale of utility of that particular class of goods. . . . Not only has one of two goods the value of the second degree of utility, but either of them has it, whichever one may choose. And three pieces have together three times the value of the third degree of utility, and four pieces have four times the value of the fourth degree. In a word, the value of a supply of similar goods is equal to the sum of the items multiplied by the marginal utility.” — Wieser.
    Do you think this analysis tenable? and do you think it inconsistent with the doctrine of total utility and consumer’s surplus?
  7. “If the modern theory of value, as it is commonly stated, were literally true, most articles of high quality would sell for three times as much as they actually bring.” What leads Clark to this conclusion? and do you accept it?

Source note: Mid-term exam from Harvard University Archives, Prof. F. W. Taussig, Examination Papers in Economics 1882-1935 (Scrapbook).

ECONOMICS 11: Final Examination [1915-16]

Arrange your answers strictly in the order of the questions. Allow time for careful revision of your answers.

  1. “The productivity of capital is, like that of land and labor, subject to the principle of marginal productivity, which is, as we have seen, a part of the general law of diminishing returns. Increase the number of instruments of a given kind in any industrial establishment, leaving everything else in the establishment the same as before, and you will probably increase the total product of the establishment somewhat, but you will not increase the product as much as you have the instruments in question. Introduce a few more looms into a cotton factory without increasing the labor or the other forms of machinery, and you will add a certain small amount to the total output…. That which is true of looms in this particular is also true of ploughs on a farm, of locomotives on a railway, of floor space in a store, and of every other form of capital used in industry.” Is this in accord with Clark’s view? Böhm-Bawerk’s? Marshall’s? Your own?
  2. What is the significance of the principle of quasi-rent for
    (a) the “single tax” proposal;
    (b) Clark’s doctrine concerning the specific product of capital;
    (c) the theory of business profits.
  3. Explain what writers use the following terms and in what senses: Composite quasi-rent; usance; implicit interest; joint demand.
  4. On Cairnes’ reasoning, are high wages of a particular group of laborers the cause or the result of high value (price) of the commodities made by them? On the reasoning of the Austrian school, what is the relation between cost and value? Consider differences or resemblances between the two trains of reasoning.
  5. “This ‘exploitation theory of interest’ consists virtually of two propositions: first, that the value of any product usually exceeds its cost of production; and, secondly, that the value of any product ought to be exactly equal to its cost of production. The first of these propositions is true, but the second is false. Economists have usually pursued a wrong method in answering the socialists, for they have attacked the first proposition instead of the second. The socialist is quite right in his contention that the value of the product exceeds the cost. In fact, this proposition is fundamental in the whole theory of capital and interest. Ricardo here, as in many other places in economics, has been partly right and partly wrong. He was one of the first to fall into the fallacy that the value of the product was normally equal to its cost, but he also noted certain apparent ‘exceptions,’ as for instance, that wine increased in value with years.” Is this a just statement of Ricardo’s view? Of the views of economists generally? In what sense is it true, if in any, that value usually exceeds cost?
  6. Explain carefully what Böhm-Bawerk means by

(a) social capital;
(b) the general subsistence fund;
(c) the average production period;
(d) usurious interest.

In what way does he analyze the relation between (b) and (c)?

  1. Suppose ability of the highest kind in the organization and management of industry became as common as ability to do unskilled manual labor is now; what consequences would you expect as regards the national dividend? the remuneration of the business manager and of the unskilled laborer? Would you consider the readjusted scale of remuneration more or less equitable than that now obtaining?
  2. What grounds are there for maintaining or denying that “profits” are (a) essentially a differential gain, (b) ordinarily capitalized as “common stock,” (c) secured through “pecuniary,” not “industrial” activity? What method of investigation would you suggest as the best for answering these questions?

 

________________________

The Distribution of Wealth (Carver)

ECONOMICS 121: Course Announcement [1916-17]

[Economics] 12. 1hf. The Distribution of Wealth. Half-course (first half-year). Tu., Th., and (at the pleasure of the instructor) Sat., at 9. Professor CARVER.

An analytical study of the theory of value and its applications, the law of diminishing utility, the nature and meaning of cost, the significance of scarcity and its relation to the general problem of social adjustment, the law of variable proportions and its bearing upon the problem of a better distribution of wealth.

ECONOMICS 121: Enrollment [1915-16]

[Economics] 12 1hf. Professor Carver. — The Distribution of Wealth.

Total 6: 3 Graduates, 1 Senior, 2 Juniors.

 

ECONOMICS 121: Final Examination [1915-16]

  1. Is there any close connection between economic value and moral value? Explain and justify your answer.
  2. How would you harmonize the Ricardian doctrine of rent with the doctrine that rent is determined by the specific or net productivity of land?
  3. What is cost and what are its leading forms at the present time? How is it related to wages, interest, and profits?
  4. What is meant by the intensive and by the extensive margins of cultivation and how are they related each to the other?
  5. Can you see any connection between the wage fund doctrine and the doctrine of non-competing groups? Explain and justify your answer.
  6. What would be the main items of your program for improving the present distribution of wealth? Give your reasons for each item.

 

________________________

Statistics: Theory, Method, and Practice (Day)

ECONOMICS 13: Course Announcement [1914-15]

[Economics] 13. Statistics: Theory, Method, and Practice. Mon., Wed., Fri., at 9. Asst. Professor DAY.

The first half of this course is intended thoroughly to acquaint the student with the best statistical methods. Such texts as Bowley’s Elements of Statistics, Yule’s Introduction to the Theory of Statistics, and Zizek’s Statistical Averages, are studied in detail. Problems are constantly assigned to assure actual practice in the methods examined.

The second half of the course endeavors to familiarize the student with the best sources of economic statistical data. Methods actually employed in different investigations are analyzed and criticized. The organization of the various agencies collecting data is examined. Questions of the interpretation, accuracy, and usefulness of the published data are especially considered.

ECONOMICS 13: Enrollment [1915-16]

[Economics] 13. Asst. Professor Day. — Statistics: Theory, Method, and Practice.

Total 10: 8 Graduates, 2 Radcliffe.

 

ECONOMICS 13: Final Examination [1915-16]

  1. Explain and criticize the following statistical table:
PER CENT OF FAMILY INCOME CONTRIBUTED BY EACH CLASS OF WORKERS BY INDUSTRIES1
Per cent of family income contributed by each class of workers in—
Cotton industry Ready-made clothing indus-try Glass indus-try Silk indus-try
New England group South-ern group
Fathers 37.7 34.0 48.4 56.0 50.5
Mothers 32.4 27.9 26.8 25.1 33.0
Male children 16 years of age and over 31.1 27.3 36.5 37.8 37.0
Female children 16 years of age and over 42.6 35.2 39.7 26.7 35.1
Children 14 and 15 years of age 18.7 22.9 14.2 18.9 16.6
Children 12 and 13 years of age 14.3 17.6 10.0 15.7 13.3
Children under 12 years of age 2 3.6 13.5

1These per cents apply only to the incomes of families having wage earners of the specified class.
2Based on incomes of two families, each having one child under 12 at work.

  1. Enumerate the means by which a bureau, charged with the administration of a state registration law, may ascertain the completeness of birth registration in any registration district.
  2. Describe and illustrate the construction of a logarithmic curve. What are the advantages and disadvantages of such a curve for the purpose of graphic presentation?
  3. What is the logical distinction, if there be any, between a weighted and a simple arithmetic mean? What are the reasons for and against weighting? Under what conditions may weighting safely be omitted?
  4. Retail price quotations for two articles are reported from fifty markets as follows:
Article A Article B
Price per dozen Number of markets reporting this price Price per bushel Number of markets reporting this price
21¢ 1 $1.00 8
22¢ 2 $1.05 12
23¢ 7 $1.10 15
24¢ 11 $1.15 10
25¢ 15 $1.25 5
26¢ 9 50
27¢ 4
28¢ 1
50

Measure by the standard deviation the relative variability in price of these two commodities. Employ the short-cut method.

  1. “Imagine an ideal republic, in some respects similar to that designed by Plato, where not only were all the children removed from their parents, but where they were all treated exactly alike. In these circumstances none of the differences between the adults could have anything to do with the differences of environments and all must be due to some differences in inherent factors. In fact, the environment correlation coefficient would be nil, whilst the heredity correlation coefficient might be high.”
    Comment upon the italicized statement.
  2. Outline a correlation study of two economic variables both of which tend to increase steadily with the growth of population, and both of which are sensitive to the fluctuations of the seasons and of the business cycle.
  3. What conditions are essential to simple sampling?
    The expected proportion of accidents per year in a certain industry is 150 per 1000 workers. A company employing 2500 workers reports 405 accidents during the year 1913. Assume that the conditions of simple sampling are met; analyze the returns to determine whether the difference between the actual and expected number of accidents is significant.

 

________________________

History and Literature of Economics to the year 1848 (Bullock)

ECONOMICS 14: Course Announcement [1914-15]

[Economics] 14. History and Literature of Economics to the year 1848. Mon., Wed., and (at the pleasure of the instructor) Fri., at 11. Professor BULLOCK.

The purpose of this course is to trace the development of economic thought from classical antiquity to the middle of the nineteenth century. Emphasis is placed upon the relation of economics to philosophical and political theories, as well as to political and industrial conditions.

A considerable amount of reading of prominent writers will be assigned, and opportunity given for the preparation of theses. Much of the instruction is necessarily given by means of lectures.

ECONOMICS 14: Enrollment [1915-16]

[Economics] 14. Professor Bullock. — History and Literature of Economics to the year 1848.

Total 14: 13 Graduates, 1 Radcliffe.

 

ECONOMICS 14: Final Examination [1915-16]

  1. What did the mercantilists teach concerning: (a) economic structure; (b) economic functions; (c) economic ideals; and (d) economic policies?
  2. At what important points does Adam Smith draw upon the works of earlier writers? What important original contributions does he make?
  3. At what points are Smith’s ideas inadequately developed or inconsistent?
  4. What important changes were made in English economic doctrines by Ricardo and Mill?
  5. Give the rest of the examination period to writing an essay upon the life, works, and economic doctrines of any economist prior to Adam Smith.

 

________________________

Analytical Sociology (Anderson)

ECONOMICS 18a1: Course Announcement [1916-17]

[Economics] 18a 1hf. Analytical Sociology. Half-course (first half-year). Tu., Th., and (at the pleasure of the instructor) Sat., at 3.30. Asst. Professor ANDERSON.

The centre of this course will be in the problems of social psychology: the raw stuff of human nature, and its social transformations; imitation, suggestion and mob-mind; the individual and the social mind; social control and the theory of social forces; the relation of intellectual and emotional factors in social life. These problems will be studied in their relations to the whole field of social theory, which will be considered in outline, with some emphasis on the influence of physiographic factors and of heredity. Leading contemporary writers will be studied, and some attention will be given to the history of social theory. Instruction will be by lectures, discussion, and reports.

ECONOMICS 18a2: Enrollment [1915-16]

[Economics] 18a 2hf. Asst. Professor Anderson. — Analytical Sociology.

Total 18: 16 Graduates, 2 Seniors.

 

ECONOMICS 18a2: Final Examination [1915-16]

  1. What is the bearing of the Mendelian theory on social problems?
  2. What difference does it make for sociology whether or not we accept the doctrine of the inheritance of acquired characters? To what extent, if at all, and in what connections, does Giddings make use of this doctrine? How far, if at all, are his conclusions incompatible with Weismann’s doctrine?
  3. Explain what is meant by the “social mind.” By “social values.”
  4. Summarize the theory of McGee as to the origin of agriculture.
  5. Compare the views of Boas and W. B. Smith as to the comparative roles of race and environment in the case of the American negro. What is your own view?
  6. What did you get from your reading of Tarde? Of Le Bon? of Ross’ Social Psychology? Let your summaries be brief, but not vague! Differentiate the books.
  7. Summarize Giddings’ chapter on Demogenic Association.
  8. Illustrate the social transformation of the raw stuff of human nature by the case of either the instinct of workmanship, the sex instinct, or the instinct of flight and hiding.
  9. What reading have you done for this course?

 

________________________

Public Finance (Bullock)

ECONOMICS 31: Course Announcement [1914-15]

[Economics] 31. Public Finance. Mon., Wed., and (at the pleasure of the instructor) Fri., at 10. Professor BULLOCK.

The course is devoted to the examination of the financial institutions of the principal modern countries, in the light of both theory and history. One or more reports calling for independent investigation will ordinarily be required. Special emphasis will be placed upon questions of American finance. Ability to read French or German is presupposed.

ECONOMICS 31: Enrollment [1915-16]

[Economics] 31. Professor Bullock. — Public Finance.

Total 16: 14 Graduates, 2 Seniors.

 

ECONOMICS 31: Final Examination [1915-16]

  1. If you were writing a treatise on public finance how far would you utilize Adam Smith’s chapter on taxation?
  2. What is Eheberg’s opinion concerning any two of the following taxes: the Ertragssteuern, the Wehrsteuer, and the property tax?
  3. What is Leroy-Beaulieu’s opinion concerning any two of the following taxes: octrois, increment taxes, and the French patente?
  4. With what different opinions concerning the incidence of the house tax are you familiar? State briefly your own opinion.
  5. Discuss the doctrine that consumption taxes tend to be “absorbed,” and state your opinion concerning the practical conclusions that follow from it.
  6. What is the incidence of the usual tax on mortgages in the United States?
  7. Compare French and British direct taxation.
  8. State the principles upon which a policy of public borrowing should be based. Should public debts be extinguished?

 

Sources:

Enrollment data: 

Harvard University. Report of the President of Harvard College 1915-1916, pp. 59-61.

Examinations (except where noted):

Harvard University. Papers Set for Final Examinations in History, History of Religions, History of Science, Government, Economics, Philosophy, Psychology, Social Ethics, Education, Fine Arts, Music in Harvard College (June, 1916), pp. 45-63.

Course Announcements: 

Division of History, Government, and Economics 1914-15 printed in Official Register of Harvard University, Volume XI, No. 1, Part 14 (May 19, 1914), pp. 62-70.

Division of History, Government, and Economics 1916-17 printed in Official Register of Harvard University, Volume XIII, No. 1, Part 11 (May 15, 1916), pp. 61-69.

Image Source:

Card catalog in Widener Library at Harvard University, ca. 1915. Library of Congress Prints and Photographs Division Washington, D.C.

Categories
Courses Gender Radcliffe

Radcliffe. Economics course offerings, 1915-1920

 

Here are six previous installments in the series “Economics course offerings at Radcliffe College”:

Pre-Radcliffe economics course offerings and Radcliffe courses for 1893-94,  1894-1900 , 1900-1905 , 1905-1910 , 1910-1915.

______________________________

 

An asterisk (*) designates Graduate courses in Harvard University, to which Radcliffe students were admitted by vote of the Harvard Faculty.

Economics
1915-16

Primarily for Undergraduates:

A. Asst. Professor DAY. — Principles of Economics.

9 Se., 20 Ju., 24 So., 1 Fr., 5 Unc., 2 Sp. Total 61

 

For Undergraduates and Graduates:

2ahf. Professor GAY.— European Industry and Commerce in the Nineteenth Century.

2 Gr., 1 Se., 2 Ju., 1 So., 1 Unc., 2 Sp. Total 9

2bhf. Professor GAY.— Economic and Financial History of the United States.

3 Gr., 2 Se., 5 Ju., 1 So., 1 Unc., 1 Sp. Total 13

6ahf. Mr. P. G. WRIGHT.— Trade-Unionism and Allied Problems.

4 Se., 1 Ju., 1 Unc. Total 6

6bhf. Mr. P. G. WRIGHT.— The Labor Movement in Europe.

4 Se., 1 Ju., 1 So., 1 Unc. Total 7

7bhf. Asst. Professor ANDERSON.— The Single Tax, Socialism, Anarchism.

1 Ju., 2 So., 1 Sp. Total 4

8ahf. Professor CARVER.— Principles of Sociology.

2 Gr., 9 Se., 12 Ju., 1 So., 1 Unc., 3 Sp. Total 28

8bhf. Asst. Professor ANDERSON.—  Principles of Sociology.

2 Gr., 2 Se., 5 Ju., 1 Unc. Total 10

Accounting

Associate Professor COLE.— Principles of Accounting.

5 Se. Total 5

Economic Theory and Method

Primarily for Graduates:

*11 Professor TAUSSIG.— Economic Theory.

1 Gr., 1 Se. Total 2

*13. Asst. Professor DAY. — Statistics. Theory, method, and practice.

1 Se. Total 1

*14. Professor BULLOCK. — History and Literature of Economics to the Year 1848.

1 Gr. Total 1

Economic History

*23. Dr. GRAS (Clark College). — Economic History of Europe to the Middle of the Eighteenth Century.

1 Gr. Total 1

Course of Research

20a. Professor GAY. — Economic History.

1 Gr. Total 1

 

Source:  Annual Report of Radcliffe College for 1915-1916Report of the Chairman of the Academic Board (September 1918), pp. 40-1.

______________________________

Economics
1916-1917

Primarily for Undergraduates:

1. A. Asst. Professor E. E. DAY.— Principles of Economics.

2 Gr., 7 Se., 23 Ju., 19 So., 1 Fr., 3 Unc., 2 Sp. Total 57

For Undergraduates and Graduates:

1ahf. Associate Professor COLE.— Accounting.

6 Se., 5 Ju., 1 Sp. Total 12

1bhf. Dr. J. S. DAVIS— Statistics.

3 Gr., 3 Se., 4 Ju., 1 Unc. Total 11

1chf. Associate Professor COLE.— Accounting (advanced course).

2 Se., 3 Ju. Total 5

2ahf. Professor GAY.— European Industry and Commerce in the Nineteenth Century.

3 Gr., 7 Se., 3 Ju., 1 Unc., 1 Sp. Total 15

2bhf. Professor GAY.— Economic and Financial History of the United States.

3 Gr., 8 Se., 6 Ju., 1 So., 1 Unc., 1 Sp. Total 20.

5. Dr. BURBANK, with lectures on selected topics by Professor BULLOCK.— Public Finance, including the Theory and Methods of Taxation.

5 Se., 3 Ju. Total 8

6ahf. Mr. P. G. WRIGHT.— Trade Unionism and Allied Problems.

3 Se., 2 Ju., 3 Unc. Total 8

6bhf. Mr. P. G. WRIGHT.— The Labor Movement in Europe.

1 Se., 2 Ju. Total 3

7. Asst. Professor ANDERSON.— Economic Theory.

3 Gr., 1 Se., 1 Ju. Total 5

8. Professor CARVER.— Principles of Sociology.

1 Gr., 4 Se., 10 Ju., 1 Unc. Total 16

Economic Theory and Method

Primarily for Graduates:

*11. Asst. Professor DAY.— Economic Theory.

1 Gr. Total 1

*12hf. Professor CARVER.— The Distribution of Wealth.

2 Gr. Total 2

Applied Economics

*34. Professor RIPLEY.— Problems of Labor.

2 Gr., 2 Se. Total 4

Course of Research

20d. Professor GAY. — Economic History.

1 Gr. Total 1

 

Source:  Annual Report of Radcliffe College for 1916-1917Report of the Chairman of the Academic Board (September 1918), pp. 91-2.

 

______________________________

Economics
1917-1918

Primarily for Undergraduates:

1. A. Asst. Professor E. E. DAY. — Principles of Economics.

1 Gr., 8 Se., 16 Ju., 29 So., 1 Fr., 7 Unc. Total 62

For Undergraduates and Graduates:

1ahf. Associate Professor COLE.— Accounting.

12 Se., 3 Ju., 3 So., 1 Unc. Total 19

1bhf. Asst. Professor E. E. DAY.— Statistics.

2 Gr., 5 Se., 3 Ju., 1 Unc. Total 11

1chf. Associate Professor COLE.— Accounting (Advanced Course).

5 Se., 1 Ju., 3 So., 1 Unc. Total 10

2ahf. Professor GAY.— European Industry and Commerce in the Nineteenth Century.

6 Gr., 6 Se., 1 Ju., 1 So., 2 Unc. Total 16

2bhf. Asst. Professor GRAS (Clark University).—Economic History of the United States.

2 Gr., 4 Se., 1 Ju. Total 7

3hf. Dr. LINCOLN.— Money, Banking, and Allied Problems.

3 Gr., 7 Se., 4 Ju., 1 So. Total 15

5. Dr. BURBANK, with lectures on selected topics by Professor BULLOCK.— Public Finance, including the Theory and Methods of Taxation.

1 Gr., 4 Se. Total 5

6ahf. Dr. LINCOLN.— Labor Problems.

2 Se., 1 Ju., 1 So. Total 4

7. Asst. Professor ANDERSON.— Theories of Social Reform.

4 Se., 1 Ju., 1 So., 1 Unc. Total 7

8. Professor CARVER.—Principles of Sociology.

2 Se., 5 Ju., 5 Unc. Total 12

Primarily for Graduates:

Accounting

Associate Professor COLE.— Accounting Problems.

1 Gr., 3 Se. Total 4

Economic Theory and Method

*11. Professors CARVER and BULLOCK.— Economic Theory.

1 Gr. Total 1

Economic History

*24hf. Professor GAY. — Topics in the Economic History of the Nineteenth Century.

1 Se. Total 1

Applied Economics

*32hf. Professor CARVER. — Economics of Agriculture.

1 Gr., 3 Se. Total 4

*34. Professor RIPLEY. —Problems of Labor.

1 Gr., 1 Se. Total 2

Course of Research

20d. Professor GAY and Asst. Professor GRAS (Clark University). — Economic History.

1 Gr. Total 1

 

Source:  Annual Report of Radcliffe College for 1917-1918Report of the Chairman of the Academic Board (January 1919), pp. 44-45.

______________________________

Economics
1918-1919

Primarily for Undergraduates:

1. A. Dr. BURBANK. — Principles of Economics.

11 Se., 30 Ju., 16 So., 1 Fr., 13 Unc. Total 71

 

For Undergraduates and Graduates:

1ahf. Professor COLE. — Accounting.

1 Gr., 6 Se., 6 Ju., 3 So. Total 16

1chf. Professor COLE. — Accounting (advanced course).

1 Gr., 2 Se., 4 Ju., 2 So. Total 9

2ahf. Dr. E. E. LINCOLN. — European Industry and Commerce in the Nineteenth Century.

1 Gr., 7 Se., 3 Ju., 1 So., 2 Unc. Total 14

2bhf. Dr. E. E. LINCOLN. — Economic History of the United States.

8 Se., 1 Ju., 1 So., 2 Unc. Total 12

3hf. Dr. E. E. LINCOLN. — Money, Banking, and Allied Problems.

1 Se., 4 Ju. Total 5

5. Dr. BURBANK, with lectures on selected topics by Professor BULLOCK. — Public Finance, including the Theory and Methods of Taxation.

3 Se. Total 3

6ahf. Dr. E. E. LINCOLN. — Trade-Unionism and Allied Problems.

5 Se., 3 Ju., 1 So. Total 9

7a. Professor BULLOCK. — Economic Theory.

9 Se., 3 Ju., 1 Unc. Total 13

8. Professor CARVER. —Principles of Sociology.

5 Se., 6 Ju., 1 So. Total 12

 

Primarily for Graduates:

Accounting

Professor COLE. — Accounting Problems.

1 Gr., 1 Se., 3 Ju., 1 So. Total 6

 

Economic Theory and Method

*13. Dr. PERSONS. — Statistics. Theory, Method, and Practice.

1 Gr., 1 Se., 1 Ju. Total 3

Applied Economics

*34. Professor RIPLEY. —Problems of Labor.

2 Se. Total 2

 

Source:  Annual Report of Radcliffe College for 1918-1919Report of the Chairman of the Academic Board (January 1920), pp. 41-42.

______________________________

Economics
1919-1920

Primarily for Undergraduates:

1. A. Asst. Professor DAY. — Principles of Economics.

9 Se., 24 Ju., 23 So., 1 Fr., 6 Unc., 2 Sp. Total 65

For Undergraduates and Graduates:

1ahf. Professor COLE.— Accounting.

2 Gr., 10 Se., 3 Ju., 2 So., 1 Unc., 1 Sp. Total 19

1bhf. Asst. Professor J. S. DAVIS.— Statistics.

9 Se., 6 Ju., 2 So., 2 Unc. Total 19

1chf. Professor COLE.— Accounting (advanced course).

1 Gr., 6 Se., 1 Ju., 2 So., 1 Sp. Total 11

2ahf. Dr. E. E. LINCOLN.— European Industry and Commerce in the Nineteenth Century.

2 Se., 1 Ju., 2 Unc. Total 5

2bhf. Dr. E. E. LINCOLN.— Economic History of the United States.

1 Gr., 6 Se., 2 Ju., 1 Unc. Total 10

3hf. Dr. E. E. LINCOLN.— Money, Banking, and Allied Problems.

4 Se., 2 Ju., 2 Unc. Total 8

4bhf. Asst. Professor DAVIS. — Economics of Corporations.

1 Gr., 6 Se., 1 Ju. Total 8

5. Asst. Professor BURBANK. — Public Finance, including the Theory and Methods of Taxation.

10 Se., 1 Ju. Total 11

6ahf. Dr. E. E. LINCOLN. — Trade-Unionism and Allied Problems.

1 Gr., 1 Se., 3 Ju., 1 Unc. Total 6

8. Professor CARVER. —Principles of Sociology.

2 Gr., 3 Se., 6 Ju., 1 So., 1 Unc. Total 13

Economic Theory and Method

Primarily for Graduates:

*11. Professor TAUSSIG. — Economic Theory.

2 Gr., 3 Se. Total 5

*12hf. Professor CARVER. — The Distribution of Wealth.

1 Gr., 2 Se. Total 3

*14. Professor BULLOCK. — History and Literature of Economics to the year 1848.

2 Gr. Total 2

Applied Economics

*32hf. Professor CARVER. — Economics of Agriculture.

1 Se. Total 1

*33hf. Professor TAUSSIG. — International Trade and Tariff Problems.

1 Gr., 1 Se. Total 2

*341. Professor RIPLEY. — Problems of Labor.

3 Gr., 4 Se., 1 Ju. Total 8

Statistics

*41. Asst. Professor DAY. — Statistics: Theory and Analysis.

2 Gr. Total.2

*42. Asst. Professor DAY. — Statistics: Organization and Practice.

2 Gr. Total 2

Course of Research in Economics

*20. Professor CARVER.

1 Se. Total 1

 

Source:  Annual Report of Radcliffe College for 1919-1920Report of the Chairman of the Academic Board (January 1921), pp. 41-42.

Image Source:  Barnard and Briggs Halls, Radcliffe College, ca. 1930-1945. Boston Public Library: The Tichnor Brothers Collection.

 

 

Categories
Harvard Suggested Reading Syllabus Undergraduate

Harvard. Principles of Economics. Course outline, readings, exam questions, 1949-50

 

Of particular interest in this two-track (for economics concentrators and non-concentrators, respectively) principles of economics course is that the Keynesian Cross chapter (XII) of Paul Samuelson’s new textbook Economics was assigned in the concentrators’ version.

The course was taught by Professor Burbank and the newly minted Harvard Ph.D. Wesley Glenn Campbell who would later be hand-picked by former President Herbert Hoover to head to the Hoover Institution.

________________________

Course Description

ECONOMICS
1949-50

Primarily for Undergraduates

Economics 1 (formerly Economics Aa and Ab). Principles of Economics

Full course. Tu., Th., Sat., at 11. This course is conducted by sections. It will be divided into sections for concentrators and for non-concentrators. There will be sections at other hours. (Radcliffe sections will meet Tu., Th., Sat., at 11 and at such other times as the enrolment may justify.) Professor BURBANK, Dr. [Wesley Glenn] CAMPBELL [Harvard Ph.D., 1948], and other MEMBERS OF THE DEPARTMENT.

Economics 1 may be taken by properly qualified Freshmen with the consent of the instructor.

Economics 1 provides an introduction to the principles required for the analysis of economic problems. The development of principles in the main fields of economics and the study of economic organization give the non-concentrator a background for the understanding of economic problems and are indispensable for the concentrator’s further work in advanced courses.

 

Source:  Harvard University Archives. Syllabi, course outlines and reading lists in Economics, 1895-2003. Box 4, Folder “Economics, 1949-1950 (1 of 3)”.

________________________

Course Enrollment

[Economics] 1 (formerly Economics Aa and Ab). Principles of Economics. (Full Co.) Professor Burbank, Dr. Campbell, and other Members of the Department.

(Fall) Total 441: 1 Graduate, 16 Seniors, 68 Juniors, 220 Sophomores, 110 Freshmen, 21 Radcliffe, 5 Special.
(Spring) Total 434: 1 Graduate, 18 Seniors, 72 Juniors, 240 Sophomores, 73 Freshmen, 26 Radcliffe, 4 Special.

 

Source: Harvard University. Annual Report of the President of Harvard College, 1949-50, p. 72.

 

________________________

ECONOMICS I—CONCENTRATORS
1949-50
First Half

Sources:

Benham and Lutz Economics, American Edition (1941)
*Bowman and Bach Economic Analysis and Public Policy, Second Edition (1949)
Burns, Neal & Watson Modern Economics(1948)
Hart, A.G. Money, Debt, and Economic Activity(1948)
Merrill, Lynch, et al, How to Read a Financial Report
*Peach and Krause Basic Data of the American Economy, Revised Edition, (1949)
Peterson, S. Economics(1949)
Schumpeter, J. A. Capitalism, Socialism, and Democracy
Slichter, S. H. Modern Economic Society(1931)
Slichter, S. H. The American Economy(1948)
Williamson, H. F. The Growth of the American Economy

*To be purchased by students.

 

PART I. Introduction

  1. The Economic Problem
    Benham: Ch. 1, General Survey
  2. Economic Institutions and Economic Development
    Burns: Ch. 2, Change and Growth in the Economy
    Bowman & Bach: Ch. 6, Economic Analysis and Public Policy

PART II. National Income, Money, Banking and Price Levels

  1. National Income
    Burns: Ch. 4, National Income and National Output
    Peach & Krause: Section I, National Income
  2. Money, Banking and Price Levels
    Merrill, Lynch, et al.: How to Read A Financial Report
    Peach & Krause: Section 4, Money and Banking
    Peterson: Ch. 10, Exchange Media. Hand-to-Hand Money
    Bowman & Bach: Ch. 10, The Banking System, the Money Supply, and Investment; Ch. 11, The Government and the Money Supply
    R.B.: Banking and Monetary Statistics, Section 10, pp. 360-366
    National Debt Series: 2, Our National Debt and the Banks; 3, Our National Debt and Interest Rates; 6, Our National Debt and Life Insurance
    Hart: Ch. 10, Inflation and Deflation

PART III. Role of Markets in the Allocation of Resources and the Determination of Relative Prices

  1. Markets—An Introduction to the Problems of Production, Distribution, Exchange and Consumption
    Bowman & Bach: Ch. 2, Income and Consumption; Ch. 3, The Economic System—A Summary View; Ch. 4, Private Enterprise, Profits, and the Price System; Ch. 5, Business Enterprise in the Modern Economy—omit appendix
  2. Price Determination and Resource Allocation
    Bowman & Bach: Book III, Production, Individual Prices, and the Allocation of Resources
    Williamson: Ch. 25, The Location of Economic Activity
    Benham: Ch. 2, Markets, pp. 38-46
    Slichter: Ch. 10, Speculative Production, pp. 215-221
  3. Public Control of Markets
    Bowman & Bach: Ch. 33, Government Policy and Business Practice
    Schumpeter: Ch. 8, Monopolistic Practices
    Peterson: Ch. 23, Market Control Policies in the United States, pp. 618-631
    Peach & Krause: Section 9, Agriculture
  4. The Productive Performance of the American Economy
    Slichter: Ch. 1, The American Economy; Ch. 6, How Good is the American Economy
    Peach & Krause: Section 2, Population and the Working Force in the United States
    Peach & Krause: Section 3, National Resources

 

Source:  Harvard University Archives. Department of Economics. Course Reading lists, syllabi, and exams 1913-1992, Box 2, Folder “Lecture Schedules and Reading Lists, 1942-1970”, Subfolder “49-55”.

________________________

ECONOMICS I—NON-CONCENTRATORS
1949-50
First Half

Sources:

Arnold, T. The Bottlenecks of Business(1940)
Benham and Lutz Economics, American Edition (1941)
Bowman and Bach Economic Analysis and Public Policy, Second Edition (1949)
*Federal Reserve System Federal Reserve Charts on Bank Credit, Money Rates and Business(Latest edition)
Hart, A.G. Money, Debt, and Economic Activity(1948)
Johnson, E. A. J. Some Origins of the Modern Economic World
Merrill, Lynch, et al., How to Read a Financial Report
*Peterson, S. Economics(1949)
Slichter, S. H. Modern Economic Society(1931)
*Slichter, S. H. The American Economy(1948)
Williamson, H. F. The Growth of the American Economy
*Wright, D. M. Democracy and Progress

*To be purchased by students.

 

PART I. Introduction

  1. The Economic Problem
    Benham: Ch. 1, General Survey
  2. Economic Institutions and Economic Development—An Historical Approach
    Johnson: Ch. 2, The Late-Medieval Background; Ch. 3, The Emergence of Capitalism; Ch. 4, The Beginnings of Scientific Technology
    Williamson: Ch. 3, The Organization of Production During the Colonial Period
    Bowman & Bach: Ch. 6, Economic Analysis and Public Policy

PART II. The Role of Markets in the Allocation of Resources and the Determination of Relative Prices

  1. A Comprehensive View of the Market System
    Peterson: Ch. 2, The Occupational and Industrial Structure; Ch. 3, Production and Income—Individual and National; Ch. 4, Framework and Problems of the Economic System
  2. The Determinants of Productive Power and the Organization of Production Under Capitalism
    Peterson: Ch. 5, Natural and Human Resources; Ch. 6, Capitalistic Production; Ch. 7, The Organization of Production; Ch. 8, Business Enterprise and the Corporate Form
    Merrill, Lynch, et al.: How to Read a Financial Report
    Peterson: Ch. 9, Finance, pp. 207-214 and 221-236
    Williamson: Ch. 14, The Capital Markets, 1789-1860; Ch. 28, The Investment Market After the War Between the States
  3. Price Determination and Resource Allocation
    Peterson: Ch. 17, The Role of Prices; Ch. 18, Supply, Demand, and Market Price
    Benham: Ch. 2, Markets, pp. 38-46
    Slichter: Ch. 10, Speculative Production, pp. 215-221
    Peterson: Ch. 19, Nature and Role of Demand and its Elasticity; Ch. 20, Cost and the Expansion and Contraction of Industries
    Williamson: Ch. 25, The Location of Economic Activity
    Peterson: Ch. 21, Output from Existing Capacity
  4. Public Regulation of Markets
    Peterson: Ch. 22, Monopoly and the Public Interest
    Williamson: Ch. 30, Industrial Concentration and Government anti-Trust Policy
    Arnold: Ch. 2, How Restraints of Trade Affect Your Standard of Living; Ch. 3, How Restraints of Trade Unbalance the National Budget; Ch. 7, Procedure under the Sherman Act; Ch. 8, The Clarification of Law; Appendix I
    Peterson: Ch. 23, Market Control Policies in the United States
    Wright: Ch. 8, The Problems of Competition
  5. The Production and Distribution of Wealth
    Slichter: Ch. 1, The American Economy; Ch. 6, How Good is the American Economy
    Wright: Ch. 7, Economic Goals and the Distribution of Wealth

PART III. Money, Banking, Price Levels and the National Income

  1. Money, Banking and Price Levels
    Peterson: Ch. 10, Exchange Media. Hand-to-Hand Money
    Bowman & Bach: Ch. 10, The Banking System, the Money Supply, and Investment; Ch. 11, The Government and the Money Supply
    R.B.: Banking and Monetary Statistics, Section 10, pp. 360-366
    National Debt Series: 2, Our National Debt and the Banks; 3, Our National Debt and Interest Rates; 6, Our National Debt and Life Insurance
    Hart: Ch. 10, Inflation and Deflation
  2. Mechanics of the International Monetary Exchange
    Benham: Ch. 26, Balance of Payments
    Hart: Ch. 15, The Foreign Exchange Market
    Benham: Ch. 27, Free Exchange Rate; Ch. 28, The Gold Standard

 

Source:  Harvard University Archives. Department of Economics. Course Reading lists, syllabi, and exams 1913-1992, Box 2, Folder “Lecture Schedules and Reading Lists, 1942-1970”, Subfolder “49-55”.

________________________

ECONOMICS I—CONCENTRATORS
1949-50
Second Half

Sources:

Benham and Lutz Economics, American Edition (1941)
*Bowman and Bach Economic Analysis and Public Policy,Second Edition (1949)
**Committee for Economic Development The Uses and Dangers of Direct Controls in Peacetime
**Economic Outlook Consumers, Workers Pay Cost of New Factories
Hart, A. G. Money, Debt, and Economic Activity(1948)
**International Bank for Reconstruction and Development Fourth Annual Report, 1948-49
**International Monetary Fund Annual Report, 1949
**Murray, P. The Steelworkers’ Case for Wages, Pensions and Social Insurance
*Peach and Krause Basic Data of the American EconomyRevised Edition (1949)
Peterson, S. Economics(1949)
Samuelson, P. Economics
Slichter, S. H. Basic Criteria Used in Wage Negotiations
**Slichter, S. H. Profits in a Laboristic Society
**Slichter, S. H. The Taft-Hartly Act
**Steel Industry Board Report to the President of the United States
**Voorhees, E. M. Statement before the Presidential Steel Board
Wright, D. M. Democracy and Progress

* To be purchased by students
**To be handed out in section meeting.

 

PART IV. The Distribution of Income

  1. Introduction
    Bowman & Bach: Ch. 28, Introduction to the Study of Income Distribution
  2. Personal Income Distribution
    Bowman & Bach: Ch. 29, Personal Income Distribution in the United States
    Wright: Ch. 7, Economic Goals and the Distribution of Wealth
  3. Determination of Returns to the Factors of Production
    Bowman & Bach: Ch. 30, Wage and Salary Income; Ch. 32, Property Income
  4. Labor Organization and Labor Markets
    Bowman & Bach: Ch. 31, The Economics of Labor Unionism
    Slichter: Basic Criteria Used in Wage Negotiations, pp. 7-31, and 36-40
    Bowman & Bach: Ch. 35, Government Policy and Labor, pp. 651-673
    Slichter, The Taft-Hartley Act
  5. The Wages, Pensions, Prices and Profits Controversy
    Economic Outlook: Consumers, Workers Pay Cost of New Factories
    Slichter: Profits in a Laboristic Society
    Murray, The Steelworkers’ Case for Wages, Pensions and Social Insurance, pp. 9-29
    Voorhees, Statement before the Presidential Steel Board
    Steel Industry Board, Report to the President of the United States, pp. 1-11

PART V. International Economic Problems

Benham: Ch. 25, The Theory of International Trade; Ch. 26, Balances of Payments
Peach & Krause: Section 5, International Trade and Finance
Hart: Ch. 15, The Foreign Exchange Market
Benham: Ch. 27, Free Exchange Rates; Ch. 28, The Gold Standard; Ch. 29, Exchange Control; Ch. 30, Import Duties and Quotas
Hart: Ch. 18, International Monetary Cooperation
International Monetary Fund: Annual Report, 1949, pp. 1-46
International Bank for Reconstruction and Development: Fourth Annual Report, 1948-49, pp. 7-37

PART VI. Public Finance and the Economic Problem

Peach & Krause: Section 6, Government Expenditures, Tax Collections, Public and Private Debt
Bowman & Bach: Ch. 36, Introduction to the Public Economy; Ch. 37, Public Expenditures; Ch. 38, Public Revenues—Taxation; Ch. 39, Taxation (Continued)
Peterson: Ch. 30, Public Policy and the Distribution of Income

PART VII. The Nature of Economic Fluctuations and Policies Directed Toward Their Control

Samuelson: Ch. 12, Saving and Investment
Peach & Krause, Review Section 1, National Income
Hart: Review Ch. 10, Inflation and Deflation
Bowman & Bach: Ch. 13, The Rate of Economic Growth; Ch. 14, Economic Fluctuations
Peach & Krause: Section 7, Price Levels and Business Fluctuations
Wright: Ch. 6, Progress and Instability
Bowman & Bach: Ch. 40, Monetary Policy and Economic Stabilization; Ch. 41, Fiscal Policy and Economic Stabilization; Ch. 42, Antimonopoly Measures, Wage-Price Policy, and Direct Controls
C.E.D.: The Uses and Dangers of Direct Controls in Peacetime

 

Source:  Harvard University Archives. Department of Economics. Course Reading lists, syllabi, and exams 1913-1992, Box 2, Folder “Lecture Schedules and Reading Lists, 1942-1970”, Subfolder “49-55”.

________________________

ECONOMICS I—NON-CONCENTRATORS
1949-50
Second Half

Sources:

Benham and Lutz Economics, American Edition (1941)
Bowman and Bach Economic Analysis and Public Policy,Second Edition (1949)
**Economic Outlook Consumers, Workers Pay Cost of New Factories
Hart, A. G. Money, Debt, and Economic Activity(1948)
**International Bank for Reconstruction and Development Fourth Annual Report, 1948-49
**International Monetary Fund Annual Report, 1949
Jewkes, J. Ordeal by Planning(1948)
*Peterson, S. Economics(1949)
*Schumpeter, J. A. Capitalism, Socialism and Democracy(1947)
*Slichter, S. H. The American Economy(1948)
Slichter, S. H. Basic Criteria Used in Wage Negotiations
**Slichter, S. H. Profits in a Laboristic Society
Sweezy, P. M. Socialism
*Wright, D. M. Democracy and Progress

* To be purchased by students
**To be handed out in section meeting.

 

PART IV. The Distribution of Income

  1. Personal Income Distribution
    Peterson: Ch. 24, Inequality—Extent and Significance; Ch. 25, Inequality in the Return from Labor
  2. Determination of Returns to the Factors of Production
    Peterson: Ch. 26, Productivity and Income; Ch. 28, The Basis of Property Incomes; Ch. 29, Profits, Interest, and Wealth
  3. Labor Organization and Labor Markets
    Bowman & Bach: Ch. 31, The Economics of Labor Unionism, pp. 492-501
    Peterson: Ch. 27, Wage-raising Policies and Practices
    Slichter: Basic Criteria Used in Wage Negotiations, pp. 7-31, and 36-40
    Bowman & Bach: Ch. 35, Government Policy and Labor, pp. 651-681
    Slichter: Ch. 2, Co-operation or Conflict in American Industry
  4. The Wages, Prices and Profits Controversy
    Economic Outlook: Consumers, Workers Pay Cost of New Factories
    Slichter: Profits in a Laboristic Society

PART V. International Economic Problems

Benham: Ch. 25, The Theory of International Trade; Review Chs. 26, 27, 28
Hart: Review, Ch. 15
Benham: Ch. 29, Exchange Control; Ch. 30, Import Duties and Quotas
Hart: Ch. 18, International Monetary Cooperation
International Monetary Fund: Annual Report, 1949, pp. 1-46
International Bank for Reconstruction and Development: Fourth Annual Report, 1948-49, pp. 7-37

PART VI. Public Finance and the Economic Problem

Bowman & Bach: Ch. 36, Introduction to the Public Economy; Ch. 37, Public Expenditures; Ch. 38, Public Revenues—Taxation; Ch. 39, Taxation (Continued)
Peterson: Ch. 30, Public Policy and the Distribution of Income

PART VII. The Nature of Economic Fluctuations and Policies Directed Toward Their Control

Peterson: Ch. 14, Total Demand and the Depression Problem; Ch. 15, Cyclical Fluctuations
Wright: Ch. 6, Progress and Instability
Slichter: Ch. 3, The Problem of Economic Stability
Wright: Ch. 11, Three Plans

PART VII. The Prospects for Economic Progress under Capitalism and Other Systems

Schumpeter: Part II, Can Capitalism Survive
Wright: Ch. 1, Science, Democracy, and Capitalism; Ch.2, The Moral Dilemma of Progress; Ch. 3, The Meaning and the Method of Democratic Progress; Ch.4, Political Democracy and the Alternatives to Competition
Schumpeter: Part III, Can Socialism Work?
Sweezy: Ch. 10, Can Socialism Provide Incentives to Work and to Efficiency?; Ch. 12, Are Socialism and Freedom Compatible?
Jewkes: Ch. 1, The Spread of Fashion; Ch.2, Is the Business Man Obsolete; Ch. 5, Confusion Among the Planners; Ch. 6, Planners as a Species; Ch. 7, Planning as a Scientific Method; Ch. 8, Planning and Prosperity; Ch. 9, Planning and Economic Stability; Ch. 10, Planning and Freedom

 

Source:  Harvard University Archives. Department of Economics. Course Reading lists, syllabi, and exams 1913-1992, Box 2, Folder “Lecture Schedules and Reading Lists, 1942-1970”, Subfolder “49-55”.

________________________

1949-50
HARVARD UNIVERSITY
ECONOMICS I
Non-Concentrators

Mid-Year Examination
January, 1950

I
(One hour and a half)
Answer both questions

  1. A member of the Board of Governors of the Federal Reserve System has recently advised Congress that the policy of the Treasury has made it impossible for the Federal Reserve authorities to use their powers as controllers of the country’s money supply. Explain carefully why Treasury and Federal Reserve policies must be coordinated and in what ways they are likely to come in to conflict. Illustrate by reference to the national debt and other problems which arose in the war and the postwar periods.
  2. The problem of the allocation of scarce resources among a multitude of possible uses is one which is largely solved automatically in our economy.
    Explain how this problem is solved. Give careful attention to the role of and inter-relationships among each of the following: consumer decisions, producer decisions and markets.

II
(One hour and a half)
Answer any THREE questions

  1. The monetary control authorities generally attempt to control the level of prices and the level of income through control of the supply of money. Using the equation of exchange as an analytic framework, analyze how a policy which changes the supply of money might work out.
  2. Answer either (a) or (b) of the following
    1. Distinguish “rate level” from “rate structure.” Discuss the criteria relied on by regulatory commissions in determining each for a public utility, noting the major problems involved.
    2. What are the major economic arguments for and against monopoly? In the light of these arguments what elements do you think should be contained in any balanced government policy toward monopoly?
  3.      aExplain the relationship between gross and net national product; between national income and aggregate personal income.
    1. Discuss a purpose for which each one of the above aggregates can be used.
    2. In the light of the above explanation and additional pertinent facts comment on the following statement: “A comparison of national income at the depth of a depression with that during a period of prosperity overstates the impact of the depression on the consuming public.”
  4. Answer TWO of the following:
    1. Explain how speculative markets control the rate of use of periodically produced goods.
    2. Restate the Malthusian thesis (law of population) using the principle of diminishing returns.
    3. Distinguish the short-run stabilization and long-run adjustment of the market for farm products. Consider both the objectives and the implied policies.
    4. Discuss the respective roles of technological change and savings and capital accumulation in the emergence of modern economic society.

Source:  Harvard University Archives. Department of Economics. Course Reading lists, syllabi, and exams 1913-1992, Box 2, Folder “Economics 1, Exams 1939-1962”.

________________________

1949-50
HARVARD UNIVERSITY
ECONOMICS I
Concentrators

Mid-Year Examination
January, 1950

 

I
(One hour and a half)
Answer both questions

  1. A member of the Board of Governors of the Federal Reserve System has recently advised Congress that the policy of the Treasury has made it impossible for the Federal Reserve authorities to use their powers as controllers of the country’s money supply. Explain carefully why Treasury and Federal Reserve policies must be coordinated and in what ways they are likely to come in to conflict. Illustrate by reference to the national debt and other problems which arose in the war and the postwar periods.
  2. Consumers’ preferences change, thus increasing the demand for a certain product which is produced under conditions of pure competition. Trace in precise fashion the results of this increase in demand on the output of the individual firms and of the industry, and on the price of the product:
    1. in the short run,
    2. in the long run.

 

II
(One hour and a half)
Answer any THREEquestions

  1. The monetary control authorities generally attempt to control the level of prices and the level of income through control of the supply of money. Using the equation of exchange as an analytic framework, analyze how a policy which changes the supply of money might work out.
  2. What are the major economic arguments for and against monopoly? In the light of these arguments what elements do you think should be contained in any balanced government policy toward monopoly?
  3. Answer TWO of the following:
    1. Discuss three important factors determining the location of economic activity.
    2. “When there is oligopoly, even without collusive agreements, price competition will tend to be ‘nonaggressive’, and price will usually be higher than otherwise.” Discuss.
    3. “Competition on a nonprice basis has become more and more important in recent years.” Discuss the effects of this trend on the allocation of resources.
    4. Discuss the process of hedging in a commodity market and its significance to the non-speculative businessman.
  4. Define Gross National Output (Product), National Income, and Income Payments (Personal Income).
    1. What is the general use of these concepts and how might each one be used specifically?
    2. How is Gross national Output related to Aggregate Demand or Expenditure?
    3. How will the relation between National Income and income Payments vary in prosperity and depression?
    4. Can we place great reliance on these concepts as measures of economic welfare?

Source:  Harvard University Archives. Department of Economics. Course Reading lists, syllabi, and exams 1913-1992, Box 2, Folder “Economics 1, Exams 1939-1962”.

________________________

1949-50
HARVARD UNIVERSITY
ECONOMICS I
Non-Concentrators

Final Examination
June, 1950

I
(One hour and a half)
Answer both questions

  1. Investment is often said to play a “strategic role” in the business cycle. What is meant by this statement? What are its implications for counter-cyclical policy?
  2. “Remuneration for labor services and a share in the social dividend are the only sources of personal income under socialism. Therefore, the socialist planners can ignore rent, interest, and profits even though they are fundamental to the functioning of a capitalist system.” Discuss.

II
(One hour and a half)
Answer both questions

  1. Without stating general conclusions as to the merits of either side, explain the basic issues involved in the dispute between labor and industry over wages, prices and profits.
  2. Discuss the elements to be considered in the establishment of a model tax system for the United States at the present level of expenditures. (This includes all levels of Government.)

III
(Thirty minutes)
Answer one question

  1. An adverse balance of payments can be corrected by (1) changes in exchange rates, (2) changes in prices and incomes, or (3) exchange and import controls.
    1. Discuss briefly how each of the above three methods may be used to correct a country’s adverse balance of payments.
    2. Discuss the extent to which the member countries of the International Monetary Fund may make use of the above three methods.
  2. Comment on the following statement: “The object of American tariff policy should be to impose sufficient duty on goods of every kind to equalize the cost of production at home and abroad.”

Source:  Harvard University Archives. Department of Economics. Course Reading lists, syllabi, and exams 1913-1992, Box 2, Folder “Economics 1, Exams 1939-1962”.

________________________

1949-50
HARVARD UNIVERSITY
ECONOMICS I
Concentrators

Final Examination
June, 1950

I
(One hour and a half)
Answer both questions

  1. Investment is often said to play a “strategic role” in the business cycle. What is meant by this statement? What are its implications for counter-cyclical policy?
  2. The establishment of product prices and of returns to factors of production are two sides of the same economic process.
    1. Analyze the forces of supply and demand which determine the return to a factor of production.
    2. Explain (in terms of producer and consumer decisions) how these returns determine and are determined by the prices of products.

II
(One hour and a half)
Answer both questions

  1. Without stating general conclusions as to the merits of either side, explain the basic issues involved in the dispute between labor and industry over wages, prices and profits.
  2. Discuss the elements to be considered in the establishment of a model tax system for the United States at the present level of expenditures. (This includes all levels of Government.)

III
(Thirty minutes)
Answer one question

  1. An adverse balance of payments can be corrected by (1) changes in exchange rates, (2) changes in prices and incomes, or (3) exchange and import controls.
    1. Discuss briefly how each of the above three methods may be used to correct a country’s adverse balance of payments.
    2. Discuss the extent to which the member countries of the International Monetary Fund may make use of the above three methods.
  2. Comment on the following statement: “The object of American tariff policy should be to impose sufficient duty on goods of every kind to equalize the cost of production at home and abroad.”

Source:  Harvard University Archives. Department of Economics. Course Reading lists, syllabi, and exams 1913-1992, Box 2, Folder “Economics 1, Exams 1939-1962”.

Image Source:  H. H. Burbank in the Harvard Class Album 1947.

Categories
Economist Market Economists Harvard Pennsylvania Williams

Harvard. Job placements of economics PhDs. Jewish candidates, 1928-29

 

In this post I provide transcriptions of four letters concerning Harvard Ph.D.s on the job market. Two of candidates (Mandell Morton Bober and Richard Vincent Gilbert) were Jewish and this was considered an important characteristic to signal to prospective employers. Nothing from the Harvard side indicates anything other than a willingness to provide information that would be revealed in the process of recruitment anyway. In an earlier post we could read a similar letter by Allyn Young’s on behalf of his protégé Arthur William Marget for a position at the University of Chicago in 1927. In the cases below we again see anti-Jewish prejudice on the demand side of the market for academic economists.

Before getting to the letters (that are also interesting for providing a glimpse into job placement at the time), I provide a bit of information about each of the Harvard alumni discussed.

______________

Harvard Ph.Ds discussed

Beach, Walter Edwards

Harvard, Ph.D. in Economics, 1929.
Thesis title: International gold movements in relation to business cycles.
A.B. Stanford University, 1922; A.M. Harvard University.
1929. Instructor in Economics and Tutor in the Division of History, Government, and Economics, Harvard University.

Bober, Mandell Morton

Harvard, Ph.D. in Economics, 1925.
Thesis title: Karl Marx’s interpretation of history.
S.B. University of Montana, 1918; A.M. Harvard University, 1920.
1925. Instructor in Economics, Boston University.
1926. Instructor in Economics. and Tutor in the Division of History, Government, and Economics, Harvard University. Cambridge, Mass.

Gilbert, Richard Vincent

Harvard, Ph.D. in Economics, 1930.
Thesis title: Theory of International Payments.
S.B. Harvard University, 1923; A.M. Harvard University, 1925.

Hohman, Elmo Paul

Harvard, Ph.D. in Economics, 1925.
Thesis title: The American whaleman: a study of the conditions of labor in the whaling industry, 1785-1885.
A.B. University of Illinois, 1916; A.M. University of Illinois, 1917; A.M. Harvard University, 1920.
1925. Assistant Professor of Economics, Northwestern University.
1926. Assistant Professor of Economics, Northwestern University. Evanston, Ill.

Patton, Harald Smith

Harvard, Ph.D. in Economics, 1926.
Thesis Title: Grain growers’ cooperation in Western Canada.
A.B. University of Toronto, 1912; A.M. Harvard University, 1921.
1926. Associate Professor of Economics, University of Cincinnati. Cincinnati, O.

Remer, Charles Frederick

Harvard, Ph.D. in Economics, 1923.
Thesis title: The foreign trade of China.
A.B. University of Minnesota, 1908; A.M. Harvard University, 1917.
1923. Instructor in Economics, and Tutor in the Division of History, Government, and Economics, Harvard University.
1926. Orrin Sage Professor of Economics, Williams College. Williamstown, Mass.

Roberts, Christopher

Harvard, Ph.D. in Economics, 1927.
Thesis title: The History of the Middlesex Canal.
S.B. Haverford College, 1921; A.M. Harvard University 1922.
1927. Instructor in Economics and Tutor in the Division of History, Government, and Economics, Harvard University.

Smith, Walter Buckingham

Harvard, Ph.D. in Economics, 1928.
Thesis title: Money and prices in the United States from 1802 to 1820.
A.B. Oberlin College, 1917; A. M. Harvard University, 1924.
1928. Assistant Professor Economics, Wellesley College.

Taylor, Overton Hume

Harvard, Ph.D. in Economics, 1928.
Thesis title: The idea of a Natural Order in Early Modern Economic Thought.
A.B. University of Colorado 1921.
1928. Instructor in Economics and Tutor in the Division of History, Government, and Economics, Harvard University.

 

Source: Harvard University. Doctors of Philosophy and Doctors of Science Who have received their Degree in Course from Harvard University, 1873-1926, with the Titles of their Theses. Cambridge: 1926. Also Annual Reports of the President of Harvard College.

______________

Carbon copy
Possible candidates for Charles Frederick Remer successor at Williams College

June 19, 1928.

Dear Professor Taussig:

Professor Burbank has asked me to write to you in answer to your letter of the 13th regarding possibilities for Remer’s position at Williams.

He believes that Bober can be recommended in the highest terms, but that the matter of his race should be mentioned. Gilbert, now at Rochester, is very able and in spite of the fact that he still has to complete his work for the Ph.D., might well be considered. He does not think so very highly of Patton; Hohman at Northwestern is fully as good.

He wonders what you would say regarding Walter Smith. He has some personal qualities that might cause trouble at Williamstown, but he is fully as capable as Remer.

If Professor Bullock has not left for Europe he suggests that he should be consulted since he knows the Williamstown situation very well.

Sincerely yours,

[unsigned, departmental secretary?]

______________

 

Carbon copy
Possible candidates for position at St. Lawrence University

January 28, 1929.

My dear Mr. Cram:

I have your note regarding the position at St. Lawrence University.

Beach probably will not go out next year. He wishes to stay here another year, and if we can make adequate provision for him we will do so.

If St. Lawrence is insistent upon the Ph.D you might recommend in very strong terms Christopher Roberts. If they will take a Jew you can recommend in superlative terms Professor M. M. Bober, now at Lawrence College; and also you might recommend under the above conditions, but perhaps less strongly R. V. Gilbert whom we expect to take the Ph.D this June.

However, before making any recommendations you should have the salary terms, the amount of teaching required, and the subjects to be taught.

Very sincerely,

H.H. Burbank.

HHB:BR

______________

Possible candidate for position at the Wharton School, University of Pennsylvania

University of Pennsylvania
Philadelphia

Wharton School of Finance and Commerce

May 16, 1929.

Professor H. H. Burbank
Department of Economics
Harvard University
Cambridge, Mass.

My dear Professor Burbank:

Thanks for your letter of May 8, informing me that Mr. Gilbert is of Jewish extraction. Professor Taussig had already told me that such was the case.

However, this will make no difference to us so long as his personality and bearing are attractive.

I am giving serious consideration to Mr. Gilbert, along with two other men who have been suggested to me from other sources. If Gilbert receives his Ph.D. this year, we may make him an offer, but we cannot consider him if he has not completed his work for the doctorate.

Sincerely yours,

[signed|
Raymond T. Bye
Acting Chairman
Department of Economics

RTB:T

______________

Possible candidate for position at the Wharton School, University of Pennsylvania (cont.)

University of Pennsylvania
Philadelphia

Wharton School of Finance and Commerce

June 17, 1929.

Professor H. H. Burbank
Department of Economics
Harvard University
Cambridge, Mass.

My dear Professor Burbank:

I hope that I did not cause you and your colleagues any inconvenience in pressing you and Dr. [O. H.] Taylor for an immediate decision on our offer to him. Things had dragged along here so long that I felt something must be done quickly and I know that I had prepared both Dr. Taylor and you for the possibility of our making him an offer, so that I felt it would not be difficult for you to make arrangements on short notice.

When I met you in Boston I was so well impressed with what you and Professor Vanderblue told me about Dr. Bober that I arranged for him to come here to meet us. We were all favorably impressed and I made every effort to secure his appointment to the position, but the Provost of the University was not willing to recommend a person of the Jewish race, so I had to give him up. It was then that I made the offer to Taylor. I think Dr. Taylor will fit into our problem for next year very nicely, for we need someone primarily to teach graduate courses. I question, however, whether we shall want to keep him permanently because, as I understand it, he is less effective as an undergraduate teacher. That is why I asked you to let him go on a year’s leave of absence. However, it is possible that the men here may like him so much that they will want to keep him permanently if he will stay. That will be for Professor E. M. Patterson to decide. He will be back as chairman of the department next year.

I want to thank you most cordially for your very material assistance in helping me to find a man to fill the vacancy here.

Sincerely yours,
[signed]
Raymond T. Bye
Acting Chairman
Department of Economics

RTB:T

 

Source:  Harvard University Archives. Department of Econoics. Correspondence & Papers 1902-1950.Box 14, Folder “Positions for 1929-30”.

Image Source: Left, Senior year picture of R.V. Gilbert and, right, tutor picture of M.M. Bober (1926) in Harvard Class Album, 1923 and 1926, respectively.

Categories
Curriculum Gender Harvard Radcliffe

Radcliffe. Economics course offerings, 1910-1915

 

Here are five more installments in the series “Economics course offerings at Radcliffe College”…

Pre-Radcliffe economics course offerings and the Radcliffe courses for 1893-94,  1894-1900 , 1900-1905 , 1905-1910 have been posted earlier.

________________

1910-1911
ECONOMICS.

Primarily for Undergraduates:—

1. Dr. HUSE and DAY. — Outlines of Economics. — Production, Distribution, Exchange, Socialism, Labor, Railroads, Trusts, Foreign Trade, Money, and Banking.

45 Undergraduates, 6 Special students. Total 51.

 

For Undergraduates and Graduates:—

3. Professor CARVER. — Principles of Sociology.—Theories of social progress. 2 hours a week, and a third hour at the pleasure of the instructor.

3 Graduates, 31 Undergraduates, 1 Unclassified student.  Total 35.
(1 Graduate, 2d half only).

6a1. Professor GAY. — European Industry and Commerce in the Nineteenth Century. Half-course. 2 hours a week, and a third hour at the pleasure of the instructor, 1st  half-year.

1 Graduate, 8 Undergraduates. Total 9.

6b2. Professor GAY. — Economic and Financial History of the United States. Half-course. 2 hours a week, and a third hour at the pleasure of the instructor, 2d half-year.

2 Graduates, 12 Undergraduates, 2 Special students, 2 Unclassified students. Total 18.

81. Dr. HUSE. — Money. A general survey of currency legislation, experience, and theory in recent times. Half-course. 3 hours a week, 1st half-year.

7 Undergraduates. Total 7.

82. Dr. DAY. — Banking and Foreign Exchange. Half-course. 3 hours a week, 2half-year.

5 Undergraduates, 1 Special student. Total 6.

14a1. Professor CARVER. — The Distribution of Wealth.  Half-course. 2 hours a week, 1st half-year.

2 Graduates, 11 Undergraduates, 2 Special students. Total 15.

14b2.  Professor CARVER. — Methods of Social Reform.—Socialism, Communism, the Single Tax, etc.  Half-course. 2 hours a week, 2half-year.

1 Graduate, 11 Undergraduates, 3 Special students, 1 Unclassified student. Total 16.

 

Primarily for Graduates:—

COURSES OF RESEARCH

20a. Professor GAY. — (a) The Millinery Trade in Boston. 1 Graduate. (b) The Small Loan Business in Boston. 1 Graduate.

Total 2.

**20b. Professor CARVER. — The Laws of Production and Valuation.

1 Graduate. Total 1.

[Note] The courses marked with two stars (**) are Graduate courses in Harvard University, to which Radcliffe students were admitted by vote of the Harvard Faculty.

 

Source:   Radcliffe College. Report of the President of Radcliffe College 1910-11, pp. 49-50.

_______________

1911-1912
ECONOMICS.

Primarily for Undergraduates:—

1. Dr. DAY and Mr. J. S. DAVIS. — Outlines of Economics. — Production, Consumption, Distribution, Exchange, Socialism, Labor Problems, Trusts, Money, Banking, and Public Finance.

43 Undergraduates, 8 Special students, 1 Unclassified student.
(1 Undergraduate, 1 Special student, 1 Unclassified student 1sthalf only.)  Total 52.

 

For Undergraduates and Graduates:—

3. Professor CARVER. — Principles of Sociology. — Theories of social progress. 2 hours a week, and a third hour at the pleasure of the instructor.

4 Graduates, 18 Undergraduates, 6 Special Students. (1 Special student, 1st half only.)  Total 28.

6a1. Professor GAY. — European Industry and Commerce in the Nineteenth Century. Half-course. 2 hours a week, and a third hour at the pleasure of the instructor, 1st  half-year.

1 Graduate, 4 Undergraduates, 3 Special students, 1 Unclassified student. Total 9.

6b2. Professor GAY. — Economic and Financial History of the United States. Half-course. 2 hours a week, and a third hour at the pleasure of the instructor, 2d half-year.

2 Graduates, 9 Undergraduates, 3 Special students. Total 14.

14a1. Professor CARVER. — The Distribution of Wealth.  Half-course. 2 hours a week, 1st half-year.

3 Undergraduates, 1 Special student. Total 4.

14b2.  Professor CARVER. — Methods of Social Reform.—Socialism, Communism, the Single Tax, etc.  Half-course. 2 hours a week, 2half-year.

3 Undergraduates, 1 Special student. Total 4.

*18. Asst. Professor COLE. — Principles of Accounting. 3 hours a week.

6 Undergraduates. (4 Undergraduates, 1st half only; 1 Undergraduate, 2half only.)  Total 6.

 

Primarily for Graduates:—

COURSES OF RESEARCH

20a. Professor GAY. — (a) The Organization of the Boot and Shoe Industry in Massachusetts in the First Half of the Nineteenth Century. 1 Graduate. (b) Economic Policy of England from 1625 to 1660. 1 Graduate. (c) Women in the Boot and Shoe Industry in Massachusetts. 2 Graduates.

Total 4.

20b. Professor CARVER. — Economic Theory.

1 Undergraduate. Total 1.

 

Source:   Radcliffe College. Report of the President of Radcliffe College 1911-12, pp. 53-54.

_______________

1912-1913
ECONOMICS.

Primarily for Undergraduates:—

1. Dr. DAY. — Outlines of Economics. — Production, Consumption, Distribution, Exchange, Socialism, Labor Problems, Trusts, Money, Banking, and Public Finance.

24 Undergraduates, 8 Special students, 4 Unclassified students.
(1 Special student, 1st half only.) Total 36.

 

For Undergraduates and Graduates:—

2a(formerly 6a1). Professor GAY. — European Industry and Commerce in the Nineteenth Century. Half-course. 2 hours a week, and a third hour at the pleasure of the instructor, 1st  half-year.

3 Graduates, 4 Undergraduates, 1 Special student. Total 8.

2b(formerly 6b2). Professor GAY. — Economic and Financial History of the United States. Half-course. 2 hours a week, and a third hour at the pleasure of the instructor, 2d half-year.

3 Graduates, 5 Undergraduates. Total 8.

7 (formerly 14). Professor CARVER. — Theories of Distribution and Distributive Justice. 3 hours a week.

9 Undergraduates, 2 Special students. Total 11.

8 (formerly 3). Professor CARVER. — Principles of Sociology.—Theories of social progress. 3 hours a week.

27 Undergraduates, 2 Special students, 2 Unclassified students. (1 Undergraduate, 1st half only.)  Total 31.

9 (formerly 18). Asst. Professor COLE. — Principles of Accounting. 3 hours a week.

5 Undergraduates. Total 5.

 

Primarily for Graduates:—

I
ECONOMIC THEORY AND METHOD

**12(formerly 13). Professor CARVER. — Scope and Methods of Economic Investigation. Half-course. 2 hours a week, 1sthalf-year.

1 Graduate. Total 1.

**13 (formerly 4). Professor RIPLEY. — Statistics, Theory, method and practice. 2 hours a week.

3 Graduates. Total 3.

II
ECONOMIC HISTORY

**23 (formerly 11). Dr. GRAY. — Economic History of Europe to 1760. 3 hours a week.

1 Special student. Total 1.

[Note] The courses marked with two stars (**) are Graduate courses in Harvard University, to which Radcliffe students were admitted by vote of the Harvard Faculty.

 

COURSES OF RESEARCH

20a. Professor GAY. — Selected Topics in Modern European Economic History.

2 Graduates. Total 4.

20b. Professor CARVER. — Economic Theory.

1 Graduate. Total 1.

 

Source:   Radcliffe College. Report of the President of Radcliffe College 1912-14, pp. 42-43.

_______________

1913-1914
ECONOMICS.

Primarily for Undergraduates:—

1. Asst. Professor E. E. DAY and Mr. BURBANK. — Principles of Economics. 3 hours a week.

33 Undergraduates, 5 Special students, 2 Unclassified students.  Total 40.

 

For Undergraduates and Graduates:—

2a(formerly 6a1). Professor GAY.— European Industry and Commerce in the Nineteenth Century. Half-course. 2 hours a week, and a third hour at the pleasure of the instructor, 1st  half-year.

1 Graduate, 10 Undergraduates, 2 Special students, 1 Unclassified student. Total 14.

2b(formerly 6b2). Professor GAY. — Economic and Financial History of the United States. Half-course. 2 hours a week, and a third hour at the pleasure of the instructor, 2d half-year.

2 Graduates, 9 Undergraduates, 1 Special student, 1 Unclassified student. Total 13.

7 (formerly 14). Asst. Professor ANDERSON. — Economic Theory: Value and Related Problems. 3 hours a week.

1 Graduate, 5 Undergraduates.  Total 6.

9 (formerly 18). Associate Professor COLE. — Principles of Accounting. 3 hours a week.

5 Undergraduates. Total 5.

 

Primarily for Graduates:—

I
ECONOMIC THEORY AND METHOD

**11. Professor TAUSSIG. — Economic Theory. Half-course. 3 hours a week.

1 Undergraduate. Total 1.

**14. Professor BULLOCK. — History and Literature of Economics to the year 1848. 2 hours a week, and a third hour at the pleasure of the instructor.

1 Graduate. Total 1.

II
ECONOMIC HISTORY

**24. Professor GAY. — Topics in the Economic History of the Nineteenth Century. Two consecutive evenings a week.

1 Undergraduate. Total 1.

 

[Note] The courses marked with two stars (**) are Graduate courses in Harvard University, to which Radcliffe students were admitted by vote of the Harvard Faculty.

 

COURSES OF RESEARCH

20a. Professor GAY. — Economic History.

2 Graduates (1 Graduate, 1st half only). Total 2.

 

Source:   Radcliffe College. Report of the President of Radcliffe College 1912-14, pp. 99-100.

_______________

1914-1915
ECONOMICS.

Primarily for Undergraduates:

1. Asst. Professor E. E. DAY. — Principles of Economics.

5 Seniors, 14 Juniors, 15 Sophomores, 1 Freshman, 3 Unclassified students, 4 Special students.  Total 42.

 

For Undergraduates and Graduates:

2ahfProfessor GAY. — European Industry and Commerce in the Nineteenth Century.

3 Graduates, 3 Seniors. Total 6.

2bhf.   Professor GAY. — Economic and Financial History of the United States

3 Graduates, 2 Seniors, 1 Junior.  Total 6.

7. Professor CARVER. — Economic Theory.

1 Graduate, 3 Seniors, 3 Juniors, 2 Sophomores.  Total 9.

8. Asst. Professor ANDERSON. — Principles of Sociology.

6 Seniors, 3 Juniors, 1 Special student. Total 10.

Accounting

Associate Professor COLE. — Principles of Accounting.

5 Seniors, 1 Junior.  Total 6.

 

Economic Theory and Method

Primarily for Graduates:

**121hf. Professor CARVER. — Scope and Methods of Economic Investigation.

1 Graduate.  Total 1.

**13. Asst. Professor DAY. — Statistics: Theory, method, and practice.

1 Graduate.  Total 1.

Applied Economics

**33 hf. Professor TAUSSIG. — International Trade, with special reference to Tariff Problems in the United States.

1 Graduate.  Total 1.

**34. Professor RIPLEY. — Problems of Labor.

1 Graduate.  Total 1.

Course of Research

20ahf. Professor GAY. — Economic History.

2 Graduates.  Total 2.

 

[Note] The courses marked with two stars (**) are Graduate courses in Harvard University, to which Radcliffe students were admitted by vote of the Harvard Faculty.

 

Source:   Radcliffe College. Report of the President of Radcliffe College 1914-15, pp. 41-42.

Image Source: From front matter of the bound version of  The Radcliffe Bulletin, 1912-13 in the Harvard University Library.

 

 

Categories
Exam Questions Harvard Suggested Reading Syllabus

Harvard. Principles of Economics, Summer School. Syllabus and Exams, 1942.

 

 

Harvard University was able to switch into a three semester per year mode in the very first summer after the U.S. entered World War II. There were two versions of the standard Principles of Economics course offered, one which extended over the twelve week summer term and one very intensive version that covered the material of a normal year-long course in just six weeks by having the students in class for two hours per days for five days per week. There was also a Principles “Lite” version that ran for only six weeks and covered just half the material apparently.

The syllabus for the full twelve week version of Economics A lists 2,600 pages of assigned reading for the  course. Nominally there would be five one-hour sessions per week, so on average for the sixty sessions students were expected to read 40-45 pages per day. Call me cynical, but I would be surprised if the average of the distribution were even half that pensum.

____________________

Summer enrollment in Principles of Economics, 1942

“The large number of course enrolments meant that individual classes were very much larger than in preceding years. The largest classes were Mathematics SAa, with 436 students, English SAa, with 347, English SAb, with 329, Mathematics SAb, with 299, and Economics SAa, with 222 students. Enrolment in 22 courses was 100 or more.”

Source:  Harvard University. Report of the President of Harvard College for 1941-1942, p. 356.

___________________

Course Announcements for Summer School 1942 

Economics SAa 1hf. Principles of Economics.
Half-course (first session). Mon. through Fri., at 11. Professor Burbank, and other members of the Department.

Economics SAa may be taken by properly qualified Freshmen with the consent of the instructor.
SAa and SAb provide an introductory study of the present organization of industry, money and the mechanism of exchange, the theory of value, foreign trade and tariff policy, the distribution of wealth; i.e., the forces governing the incomes of the laboring, land-owning, capitalist and business classes, and the relation of government to industry. The course is conducted entirely by oral discussion.

 

Economics SAb 2hf.Principles of Economics
Half-course (second session). Mon. through Fri., at 11. Professor Chamberlin, and other members of the Department.

Economics SAb may be taken by properly qualified Freshmen with the consent of the instructor.
Economics SAa is a prerequisite for the course.
For description see SAa.

 

Economics SA1(to count as a whole course in the first session). Principles of Economics
Whole course (first session). Mon. through Fri., 9 to 11.  Professor Burbank, and other members of the Department.

Economics SAis identical with SAa and Sab, the two, however, combined and completed in one session. Freshmen will not be admitted to this course. For description see SAa.

 

Economics SB 1hf. Principles of Economics
Half-course (first session). Mon. through Fri., at 11. Dr. Monroe.

If a Harvard student counts Economics SB for a degree, Economics may be counted as a half-course only. Ordinarily students concentrating in History, Government, and Economics must take Economics A, SA, or SAa and SAb.
Course SB gives a general introduction to economic study, and a general view of Economics for those who have not further time to give to the subject.

 

Source:   Final Announcement of the Courses of Instruction offered in the Summer Term 1942 published in Official Register of Harvard University, Vol. 39, No. 16 (April 20, 1942), pp. 21-22.

____________________

ECONOMICS A
Summer Term, 1942

Sources: Arnold, Thurman The Bottlenecks of Business (1940)

*

Benham and Lutz Economics, American Edition (1941)
Bidwell, P. Economic Defense of Latin America (1941)

**

Federal Reserve System Federal Reserve Charts on Bank Credit, Money Rates and Business (1941)
Garver and Hansen Principles of Economics, Revised Edition (1937)
Golden and Ruttenberg The Dynamics of Industrial Democracy (1942)
Johnson, E.A.J. Some Origins of the Modern Economic World (1936)

**

Luthringer, Chandler and Cline Money, Credit and Finance (1938)
Meyers, A.L. Elements of Modern Economics (1937)

**

Neal, A.C., Editor Introduction to War Economics (1942)
Slichter, S.E. Modern Economic Society (1928)

-ditto-

The Economics of Collective Bargaining (reprint)

-ditto-

The Period 1919-1936 in the United States, Its Significance for Business Cycle Theory, in Review of Economic Statistics, Vol. XIX, Feb. 1937, No. 1, Part I

**

Staff members Syllabus: Economics A
Taussig, F.W. Principles of Economics, Vol. I Third Edition Revised (1921)

**

Taylor, H. Main Currents in Modern Economic Life (1941)
T.N.E.C. Price Behavior and Business Policy, Monograph No. 1 (1941)
T.N.E.C. Competition and Monopoly in American Industry, Monograph No. 21 (1940)

** To be purchased by students
* Suggested for purchase

Note:  Essay due at end of eight week.

 

ECONOMICS A
Outline and Reading Assignments
Summer Term, 1942

 

Weeks Pages
1st Part I. EMERGENCE OF MODERN ECONOMIC INSTITUTIONS
The economic problem; historical development of social and legal institutions; their effect on the economic problem.
Johnson,  Ch. 1, Economic Activity and Economic Development 7
_______, Ch. 2, The Late-Medieval Background 21
_______, Ch. 3, The Emergence of Capitalism 34
_______, Ch. 4, The Beginnings of Scientific Technology 32
_______, Ch. 5, The Formulation of Capitalist Theory 23
_______, Ch. 6, Protection and the Transplantation of Industrialism 24
_______, Ch. 7, The Export of Capital and the Genesis of Economic Imperialism 15
156
Part II. MODERN ECONOMIC INSTITUTIONS
A—The economic problem again; how it is solved today; the concept of useful production.
Benham, Ch. 1, General Survey 17
Taussig, Ch. 2, Of Labor in Production 13
30
B—Description of money flows and goods flows in a capitalist society; the relation of the division of labor to these flows; the forms of business organization and their relation to the division of labor.
Taylor, Ch. 6, Vol. I, The National Income and its Distribution 18
2nd _______, Ch. 12, Vol. I, Industrial Techniques 16
Taussig, Ch. 3, Division of Labor 18
_______, Ch. 4, Large Scale Production 15
Slichter, Ch. 8, (M.E.S.) Modern Business Organization 26
93
Part III. THE DIVISION OF LABOR AND MONEY
Division of Labor necessitates exchange; exchange is facilitated by the use of money; digression to explain the working of the monetary system in the United States.
Luthringer, Ch. 1, Functions and Significance of Money 24
_______, Ch. 2, Kinds of Money 23
_______, Ch. 3, Credit and Credit Instruments 15
_______, Ch. 4, Investment Institutions and Commercial Banking 23
Pamphlet, Credit Expansion, in Economics A Syllabus 14
Luthringer, Ch. 5, Central Banking and the Federal Reserve System 20
_______, Ch. 6, The Quantitative Control of Bank Credit 18
3rd _______, Ch. 7, Meaning of the Value of Money 14
_______, Ch. 8, Equation of Exchange and the Quantity of Money 16
_______, Ch. 9, Velocity of Money and the Volume of Trade 17
184
Part IV. THE SOLUTION TO THE ECONOMIC PROBLEM
 

 

 

 

 

 

 

 

 

 

 

 

 

 

 

A—The Markets for Commodities
Analysis in this section refers only to the determination of prices and quantities of commodities produced by a business firm. No answers are given to the questions: Why are wages, rents, interest high or low?
A.1 The Business Firm vis-à-vis Consumers in an unregulated market
A.1.a Consumer Demand
Why consumers spend their incomes as they do, the process whereby consumer demand is transmitted to the market.
Benham, Ch. 2, Markets 25
_______, Ch. 3, Demand 16
_______, Ch. 4, Prices with a Fixed Demand 10
_______, Ch. 5, Changes in Demand 10
61
A.1.b The Business Firm
The final relationship of cost to price depends on the competitive conditions in each commodity market; the profit motive the main determinant of the firm’s decisions.
Benham, Ch. 12, The Controlling Power of Demand 8
_______, Ch. 13, The Problems of the Firm (omit Sec. 11) 37
_______, Ch. 14, Monopoly 18
Monograph 21, Ch. 1, the Nature and Significance of Competition 18
Monograph 1, Part 1, Ch. 2, Nonprice Competition 53
4th _______, 1, Part 2, Ch. 1, Types of Geographical Price Structures 14
148
A.2 Effect of Government Regulation on the unregulated market
A.2.a Introduction
General analysis of regulation and impediments to free markets. Sections b, c, and d elaborate more completely some issues presented here.
Arnold, Ch. 1, The Basic Problem of Distribution 19
_______, Ch. 2, How Restraints of Trade Affect your Standard of Living 25
_______, Ch. 3, How Restraints of Trade Unbalance the National Budget 13
_______, Ch. 4, A Free market in time of National Emergency or War 30
_______, Ch. 5, An Elastic Procedure…to Prevent…Seizure of…Power… 24
_______, Appendix I and II 20
_______, Ch. 6, The Test is Efficiency and Service—not size 15
_______, Ch. 7, Procedure Under the Sherman Act… 31
5th _______, Ch. 8, The Clarification of Law through public enforcement 26
_______, Ch. 9, Antitrust Enforcement for the Betterment of the Consumer 21
_______, Ch. 10, Bottlenecks between the Farm and the Table 26
_______, Ch. 11, Labor—Restraints of Trade among the Underdogs 19
_______, Ch. 12, The Rise of a Consumer Movement 37
306
A.2.b Regulation and the Consumer
Taylor, Vol. II, Ch. 37, Consumption Standards 15
____________, Ch. 38, Consumers and the Business System 19
____________, Ch. 39, Consumer Cooperation 20
54
6th A.2.c Regulation and the Business Firm
(1) Monopoly
Taylor, Vol. I, Ch. 15, The Growth of Big Business 18
___________, Ch. 16, The Trend Toward Monopoly 17
___________, Ch. 17, Monopolies and Public Policy 16
51
(2) Public Utilities
Techniques of regulation different for firms classified as public utilities; the TVA—an example of a new regulatory device.
Taylor, Vol. II, Sec. 13, pp. 363-364 Introduction 2
____________, Ch. 48, The Nature and Scope of Public Regulation 15
___________, Ch. 49, The Price of Utility Services 16
___________, Ch. 50, Recent Expansion of Federal Control 19
___________, Ch. 51, The State as Operator 17
69
A.2.d Agriculture—a special problem
Taylor, Vol. II, Sec. 8, Agriculture and the Market 3
____________, Ch. 30, The American Farmers 18
____________, Ch. 31, Farmers in the Market System 19
____________, Ch. 32, Agriculture and Public Policy 24
64
7th B. The Markets for Productive Agents (Factors of Production)
The Analysis in this section refers to the determination of the prices of the factors of production, land, labor, capital and entrepreneurship in the markets where they are bought and sold. The entrepreneur’s reward, profit, is decided for him by the success or failure of his production plan. This market is, of course, not independent of the commodity markets. The unclassified reading discusses the productive agents of the United States:
Taylor, Vol. I, Ch. 8, How Productive Resources are Used 17
___________, Ch. 9, Population 19
___________, Ch. 10, Land 15
___________, Ch. 11 Localized Natural Resources 21
___________, Ch. 13 Capital 19
91
B.1 The Factors vis-à-vis Firms in an Unregulated Market
B.1.a. The Pricing Process in General
Benham, Ch. 9, Combination of Factors (omit sec’s 5,6) 18
Meyers, Ch. 11, The Distribution of Income 11
29
B.1.b. The Prices of Each Factor of Production
Benham, Ch. 15, The Mobility of Factors of Production 10
_______, Ch. 16, Wages, pp. 258-269 20
_______, Ch. 18, Rent 13
Garver and Hansen, Ch. 26, Interest 23
_______________, Ch. 27, Profits 12
78
8th B.2 Effects of Government Regulation and other Institutional Aspects of Distribution on the Markets for Factors.
B.2.a The Labor Market
Taylor, Vol. II, Ch. 33, The American Labor Market, pp. 75-89 14
___________, Ch. 35, The Labor Movement 24
Slichter (pamphlet). The Economics of Collective Bargaining 23
Taylor, Vol. II, Ch. 36, Public Policy Regarding Labor 22
Benham, Ch. 16, Wages, pp. 269-275 (section 9) 6
Golden, entire book. Write an essay of not more than 1200 words evaluating the ideas in the book. 347
436
9th B.2.b The Market for Savings
It is to be noted that firms and others may secure funds from credit created by commercial banks.
Taylor, Vol. I, Ch. 23, pp. 431-434 only (self financing by corporations) 4
__________, Ch. 24, Investment Credit Institutions 19
__________, Ch. 25, The Security Markets 17
__________, Ch. 26, Regulation of Securities and Exchanges 18
58
C. Public Finance and the Economic Problem
The State not only regulates markets as described above but also influences the prices of factors and commodities in the process of financing the production of public goods (roads, protection, etc.). The effects of government finance on the level of national income to be postponed to Part VI.
Luthringer,  Ch. 12, The Public Economy 13
_________, Ch. 13, The Revenue System 20
_________, Ch. 14, Tax Incidence 26
_________, Ch. 15, The Income Tax 20
_________, Ch. 16, Property and Other Taxes 18
_________, Ch. 17, Public Credit 14
_________, Ch. 18, Principles of Public Credit 16
127
Part V. INTERNATIONAL ASPECTS OF MARKETS AND FINANCE
Analysis of international trade and finance; the free market and the regulated market. Previous analysis emphasized only domestic markets although the principle of international trade is applicable to regions within a country to some extent; Latin America and the War.
10th Benham,  Ch. 25, The Theory of International Trade 22
_______, Ch. 26, Balances of Payments 10
_______, Ch. 27, Free Exchange Rates 10
_______, Ch. 28, The Gold Standard 22
_______, Ch. 29, Exchange Control 8
_______, Ch. 30, Import Duties and Quotas 9
Bidwell, Ch. 1, War and the Monroe Doctrine 16
______, Ch. 2, Propaganda and Politics 13
______, Ch. 3, German Economic Penetration 12
______, Ch. 4, The Weapons of Economic Defense 33
______, Ch. 5, The Fallacy of Hemisphere Self-Sufficiency 14
169
Part VI. PROSPERITY AND DEPRESSION
Analysis of the reasons why all prices move together; why all factors tend to become unemployed—hence the emphasis on the movements of national income. Previously, the analysis of prices was chiefly concerned (with the exception of the value of money) with relative prices. Existence of international markets tends to spread the cyclical pattern.
A. Explanation and Verification of Business Cycle Theories
Benham, Ch. 7, The Volume of Production, pp. 113-125 13
Garver and Hansen, Ch. 21, Business Cycles 18
11th Benham, Ch. 20, Economic Progress and the Trade Cycle, pp. 347-356 9
Slichter, R.E.S., The Period 1919-1936 in the U.S. Its Significance for Business Cycle Theory 19
59
B. Governmental Policy and Business Cycles
Taylor, Vol. II,  Ch. 44, Deficit Spending 16
____________, Ch. 12, The State as Provider (Introduction) 2
____________, Ch. 45, Providing Minimum Needs 17
____________, Ch. 46, Social Security 19
____________, Ch. 47, Public Housing 17

71

Part VII. TOTALITARIAN ALTERNATIVES TO CAPITALISM
Analysis of the solution to the economic problem in totalitarian economic systems.
Taylor, Vol. II, Sec. 14, Totalitarian Alternatives to Free Markets (Introduction) 2
____________, Ch. 52, Economic Basis of Totalitarianism 16
____________, Ch. 53, The Soviet Economy 23
____________, Ch. 54, The Fascist Economy in Italy 19
____________, Ch. 55, The National Socialist Economy in Germany 20
80
12th Part VIII. THE ECONOMICS OF WAR
Neal,  Ch. 1, Basic Economic Problems of War 15
____, Ch. 2, Economic War Potential 23
____, Ch. 3, Problems of War Production 21
____, Ch. 4, War Labor Problems 25
____, Ch. 5, Financing the War Effort 19
____, Ch. 6, Financing the War Effort, Business Finance 27
____, Ch. 7, Wartime Management of the Monetary and Banking System 28
____, Ch. 8, The Control of Individual Prices 31
____, Ch. 9, Economic Warfare 20
____, Ch.10, Post-War International Economics 19
228

 

Source: Duke University. David M. Rubenstein Rare Book and Manuscript Library. Economists’ Papers Archives. Randall Hinshaw Papers, Box 1, Folder “Schoolwork, 1940s”.

____________________

Final examination first half of course

SUMMER SCHOOL OF HARVARD UNIVERSITY
[First Session. Summer, 1942]
ECONOMICS SAa

Part I

(One hour)

Write on BOTH of the following in this section:

  1. Analyze carefully the effects of the following on average cost, marginal cost, marginal revenue and price for the individual firm in the short run:
    1. A tax of ten cents per unit of output on a monopolist.
    2. A subsidy of ten cents per unit of output to firms in a purely competitive industry.
  2. Trace the repercussions on member bank reserve balances, Treasury deposits with the Federal Reserve System and reserves of the Federal Reserve System caused by:
    1. The purchase and sterilization of gold by the Treasury.
    2. The purchase of U.S. government bonds by the Federal Reserve System.
    3. A transference of Treasury deposits from the Federal Reserve System to member banks.

Part II

(Two hours)

Write on any FOUR of the following in this section:

  1. “Regulation of public utility rates may be effected by limiting charges to yield a fair return on a fair value of the property. This will result in prices comparable to competitive conditions. An alternative type of rate regulation may be accomplished by using as a ‘yardstick’ the rates which a government-owned plant could economically charge.”
    1. Does the use of the fair return formula approximate the price which would evolve in a competitive market for the same commodity?
    2. May the rates fixed by the Tennessee Valley Authority be used as a “yardstick” for privately-owned power companies?
  2. “There seems to be a common belief that banks, by some process of sleight of hand, contrive to create a multiple of the amount of money they receive. The truth is that they can lend not more, but less than the amount of money that comes into their hands.”
    Do you agree? Explain fully.
  3. “It is not size in itself that we want to destroy….What ought to be emphasized is…the evil of industries which are not efficient or do not pass (the gains from) efficiency on to consumers.” Arnold.
    Examine the consistency of this statement.
  4. Contrast carefully the industrial economic world of nineteenth century England with the industrial economic world of fourteenth century England.
  5. Write a letter to your congressman briefly explaining what you believe to be the basic American farm problems and critically evaluating the New Deal attempts to alleviate them.

Source:  Harvard University Archives. Department of Economics, Course reading lists, syllabi, and exams 1913-1992 (UA V 349.295.6), Box 1, Folder “Economics 1, Exams 1939-1962”.

____________________

Final examination second half of course

HARVARD UNIVERSITY
[Second Session. Summer, 1942]
ECONOMICS SAb

Part I

(One hour)

Write on BOTH of the questions in this section:

  1. “The main fiscal problem of the war is the diversion of a large share of the national income from the private economy to the public fisc for war purposes.” Outline and defend a plan of taxation and borrowing which in your opinion effectively will solve this problem.
  2. Analyze the effects of an increase in the supply of labor on (a) the remuneration of the various productive factors and (b) the changes in output of those industries whose costs are mainly labor and those whose costs are mainly capital.

Part II

(Two hours)

Write on any FOUR of the questions in this section:

  1. Write an essay on the topic of cyclical unemployment emphasizing (a) the processes by which full employment is supposed to be effected in a free enterprise economy and (b) the reasons why these processes have failed to operate.
  2. What controls are necessary for the orderly and equitable distribution of goods during war time in the markets for factors of production and the markets for consumers’ commodities? Indicate the results likely to follow from partial rather than complete controls in these two major groups of markets.
  3. “Labor unions cannot raise the wages of labor within an occupation without reducing the number employed in that occupation since the entrepreneur cannot afford to pay labor more than the value of its marginal product.” Do you agree? Explain fully.
  4. Discuss carefully three methods of correcting an adverse balance of payments. Indicate the effects of each method on the level of domestic money incomes, the foreign exchange rate, merchandise exports and imports and short-term capital movements.
  5. Explain the chief methods of regulating securities markets in the United States. State concisely the functions of securities markets and evaluate the success of regulation in aiding the orderly functioning of these markets.

Source:  Harvard University Archives. Department of Economics, Course reading lists, syllabi, and exams 1913-1992 (UA V 349.295.6), Box 1, Folder “Economics 1, Exams 1939-1962”.

____________________

Final Exam Intensive Course, First Session 1942

SUMMER SCHOOL OF HARVARD UNIVERSITY
[First Session. Summer, 1942]
ECONOMICS SA1

I

(One hour)

  1. Reply fully to the following questions:
    1. Which are the main problems confronting the economy of this country during the present war?
    2. What measures of economic policy would you propose to cope with them?

II

(About one half hour each)

  1. What are the effects of the practice of self-financing by corporations upon
    1. the rate of interest;
    2. the allocation of the nation’s resources;
    3. the prevalence of competition, or of monopoly, in the economy as a whole?
  2. Let a tax be imposed upon a monopolist and the amount due be determined by either of the following methods:
    1. a fixed percentage of his profits;
    2. a fixed money amount per unit of output;
    3. a fixed percentage of the total money value of his sales;
    4. a global fixed sum, independent of either output or sales.

In which of these cases will he be able to shift the tax forward? Prove your conclusions by a graphical analysis.

  1. Criticize the following statement carefully:
    “The average citizen is inclined to think that there is nothing at all which he himself can do to check inflation. He considers the anti-inflationary fight a task for Uncle Sam only and therefore urges control of prices and labor and the draining off of excessive purchasing power. Actually, the citizen himself can do a great deal, and the efforts of the government will be far less effective unless he does.
    “He can co-operate with the government by putting money in the bank and making it work for him, instead of drawing it out, letting it lie idle and exposing it to the danger of theft, fire and forgetfulness, He can pay his debts. He can discharge his mortgage more rapidly. He can make larger down payments on installment purchases than he has to. He can be more generous to poor relatives. He can see his oculist, dentist or family doctor more often, and pay cash. He can contribute more liberally to local and national charities. He can refuse to hoard goods. He can refrain from rushing to buy the very articles of which there is a shortage….” (The Boston Herald, July 28, 1942, p. 14)
  2. Trace the main effects that the abolition of the tariff on beef (one of the major export articles of Argentina) would, in peacetime, have upon the economies of the United States and of Argentina. Distinguish the short-run and the long-run consequences.

Source:  Harvard University Archives. Harvard University. Final examinations, 1853-2001. (HUC 7000.28) Box 6, Papers Printed for Summer Examinations First Session, August, 1942.

____________________

Final Examination for Principles of Economics “Lite”

SUMMER SCHOOL OF HARVARD UNIVERSITY
[First Session. Summer, 1942]
ECONOMICS SB

I

(About one hour)

  1. Write an essay on one of the following topics:
    1. Long-run value under competition,
    2. The rate of wages,
    3. Investment and interest,
    4. Profits

II

(Answer TWO questions from this group.)

  1. How should you expect the following to affect the selling price of a farm: an increase in population; a fall in the rate of interest; the opening of a new market for its products; a bad crop failure?
  2. Explain the meaning of the following terms and show how one of them enters into the explanation of economic phenomena: comparative advantage; bank reserves; the gold points; marginal revenue.
  3. What are the principal factors responsible for cyclical fluctuations in business activity?

III

(Answer TWO questions from this group.)

  1. Explain the nature and operation of the forces which cause variations in the purchasing power of the dollar.
  2. Outline the principal forms of unemployment and discuss one of them in some detail.
  3. Compare the advantages and disadvantages of the different methods available for financing a war.

Source:  Harvard University Archives. Harvard University. Final examinations, 1853-2001. (HUC 7000.28) Box 6, Papers Printed for Summer Examinations First Session, August, 1942.

Image Source: Harvard’s Commencement in 1943. From The Harvard Gazette, November 10, 2011.